You are on page 1of 67

Certificate Mathematics in Action Full Solutions 4B

9 More about Trigonometry (II)


• • • • • • • • • • • • • • • • • • • • • • • • • • • • • • • • • • • • •

Activity By the sine formula,


BC AC
=
sin A sin B
Activity 9.1 (p. 202)
23 cm x cm
=
1. (a) Yes sin 53° sin 76°
(b) YP 23 sin 76°
x=
FD XY sin 53°
(c) Slope of PF = , slope of PX = ,
DP YP = 27.9 (cor. to 3 sig. fig.)
EC
slope of PE =
CP A + B + C = 180°
4. 48° + B + 62° = 180° (∠ sum of △)
2. PX. Since FD = XY = EC and YP is the shortest in
length among line segments between AB and DC, B = 70°
XY By the sine formula,
therefore has the greatest value.
YP a AB
=
sin A sin C
Follow-up Exercise a
=
5 cm
sin 48° sin 62°
p. 168 5 sin 48°
a= cm
sin 62°
Sides Angle Area = 4.21 cm (cor. to 3 sig. fig.)
1. a = 4 cm, b = 6 cm C = 30° 6 cm2 b AB
=
2. a = 5 cm, b = 8 cm C = 45° 14.1 cm2 sin B sin C
3. b = 7 cm, c = 9 cm A = 102° 30.8 cm2 b 5 cm
=
4. a = 7 cm, c = 9 cm B = 110° 29.6 cm2 sin 70° sin 62°
5. a = 12 cm, c = 9 cm B = 90° 54 cm2 5 sin 70°
b= cm
sin 62°
p. 175 = 5.32 cm (cor. to 3 sig. fig.)
1. By the sine formula,
AB AC
= A + B + C = 180°
sin C sin B 5. (∠ sum of △)
28° + 115° + C = 180°
x cm 8 cm
= C = 37°
sin 30° sin 43°
8 sin 30° By the sine formula,
x=
sin 43° a AB
=
= 5.87 (cor. to 3 sig. fig.) sin A sin C
a 9 cm
=
A + B + C = 180° sin 28° sin 37°
2. (∠ sum of △) 9 sin 28°
70° + B + 54° = 180° a= cm
sin 37°
B = 56°
= 7.02 cm (cor. to 3 sig. fig.)
By the sine formula,
AB AC b AB
= =
sin C sin B sin B sin C
x cm 7 cm b 9 cm
= =
sin 54° sin 56 ° sin 115° sin 37°
7 sin 54° 9 sin 115°
x= b= cm
sin 56° sin 37°
= 6.83 (cor. to 3 sig. fig.) = 13.6 cm (cor. to 3 sig. fig.)

A + B + C = 180° A + B + C = 180°
3. (∠ sum of △) 6. 60° + 50° + C = 180° (∠ sum of △)
A + 76° + 51° = 180°
A = 53° C = 70°

1
9 Basic Properties of CirclesMore about Trigonometry (II)

By the sine formula, 2. By the sine formula,


b BC a b
= =
sin B sin A sin A sin B
b 10 cm 10 cm 8 cm
= =
sin 50° sin 60° sin 35° sin B
10 sin 50° 8 sin 35°
b = cm sin B =
sin 60° 10
= 8.85 cm (cor. to 3 sig. fig.) = 0.4589
c BC B = 27.3° (cor. to 3 sig. fig.) or 152.7° (rejected)
=
sin C sin A A + B + C = 180° (∠ sum of )
c 10 cm 35° + 27.314° + C = 180°
=
sin 70° sin 60° C = 118° (cor. to 3 sig. fig.)
10 sin 70°
c= cm By the sine formula,
sin 60°
c a
= 10.9 cm (cor. to 3 sig. fig.) =
sin C sin A
c 10 cm
p. 178 =
sin 117.69° sin 35°
1. By the sine formula,
10 sin 117.69°
a b c= cm
= sin 35°
sin A sin B
7 cm 9 cm = 15.4 cm (cor. to 3 sig. fig.)
=
sin 43° sin B
9 sin 43° 3. By the sine formula,
sin B =
7 a b
=
= 0.8769 sin A sin B
B = 61.3° (cor. to 3 sig. fig.) or 6 cm 8 cm
=
sin 130° sin B
119° (cor. to 3 sig. fig.) 8 sin 130°
sin B =
When B = 61.3°, 6
A + B + C = 180° (∠ sum of ) = 1.02 (cor. to 3 sig. fig.)
43° + 61.265° + C = 180° ∵ sin B ≤ 1 for all values of B.
C = 75.7° (cor. to 3 sig. fig.) ∴ There are no solutions for B.

By the sine formula, 4. By the sine formula,


c a a b
= =
sin C sin A sin A sin B
c 7 cm 6 cm 12 cm
= =
sin 75.735° sin 43° sin 30° sin B
7 sin 75.735° 12 sin 30°
c = cm sin B =
sin 43° 6
= 9.95 cm (cor. to 3 sig. fig.) =1
When B = 119°, ∴ B = 90°
A + B + C = 180° (∠ sum of ) A + B + C = 180° ∠
( sum of △)
43° + 118.73° + C = 180° 30° + 90° + C = 180°
C = 18.3° (cor. to 3 sig. fig.) C = 60°
By the sine formula, By the sine formula,
c a c a
= =
sin C sin A sin C sin A
c 7 cm c 6 cm
= =
sin 18.265° sin 43° sin 60° sin 30°
7 sin 18.265° 6 sin 60°
c= cm c = cm
sin 30°
sin 43°
= 10.4 cm (cor. to 3 sig. fig.)
= 3.22 cm (cor. to 3 sig. fig.)

2
Certificate Mathematics in Action Full Solutions 4B

p. 185 a 2 + c2 − b2
1. (a) By the cosine formula, cos B =
2ac
b 2 = a 2 + c 2 − 2ac cos B 40 2 + 73.410 2 − 552
=
x = 17 2 + 20 2 − 2(17)( 20) cos 125° 2( 40)( 73.410)
= 32.8 (cor. to 3 sig. fig.) B = 47.5° (cor. to 1 d.p.)

(b) By the cosine formula, A = 180° − 100° − 47.547°


c 2 = a 2 + b 2 − 2ab cos C = 32.5° (cor. to 1 d.p.)

x = 252 + 22 2 − 2( 25)( 22) cos 42°


= 17.1 (cor. to 3 sig. fig.) p. 189
a+b+c
1. Let s = , where a = 14 m, b = 8 m and c = 10 m.
2
2. (a) By the cosine formula, 14 + 8 + 10
s = m
a 2 + b2 − c 2 2
cos θ = ∴
2ab = 16 m
182 + 14 2 − 112 s − a = (16 − 14) m = 2 m
=
2(18)(14) s − b = (16 − 8) m = 8 m
133
= s − c = (16 − 10) m = 6 m
168
θ = 37.7° (cor. to the nearest 0.1°) = s ( s − a )( s − b)( s − c )
∴ Area of △ABC = 16 × 2 × 8 × 6 m 2
(b) By the cosine formula,
a 2 + c 2 − b2 = 39.2 m 2 (cor. to 3 sig. fig.)
cos θ =
2ac
18 + 17 2 − 312
2
a +b+c
= 2. Let s = , where a = 33 m, b = 29 m and c = 24 m.
2(18)(17) 2
29 32 + 29 + 24
= −
51 ∴ s = 2
m
θ = 124.7° (cor. to the nearest 0.1°) = 43 m
s − a = ( 43 − 33) m = 10 m
3. By the cosine formula, s − b = (43 − 29) m = 14 m
b2 + c 2 − a 2 s − c = ( 43 − 24) m = 19 m
cos A =
2bc = s ( s − a )( s − b)( s − c )
26 + 352 − 282
2
∴ Area of △ABC = 43 × 10 × 14 × 19 m 2
=
2( 26)(35)
= 338 m 2 (cor. to 3 sig. fig.)
1117
=
1820
A = 52° (cor. to the nearest degree) a+b+c
3. Let s = , where a = 26 m, b = 37 m and c = 21 m.
2
a 2 + c 2 − b2 26 + 37 + 21
cos B =
2ac ∴ s = 2
m
282 + 352 − 26 2 = 42 m
=
2( 28)(35) s − a = ( 42 − 26) m = 16 m
1333 s − b = (42 − 37) m = 5 m
=
1960 s − c = ( 42 − 21) m = 21 m
B = 47° (cor. to the nearest degree)
= s ( s − a )( s − b)( s − c )
C = 180° − A − B ∴ Area of △ABC = 42 × 16 × 5 × 21 m 2
∴ = 180° − 52.14° − 47.15°
= 266 m 2 (cor. to 3 sig. fig.)
= 81° (cor. to the nearest degree)

a+b+c
4. By the cosine formula, 4. Let s = , where a = 17 m, b = 27 m and c = 17 m.
2
c 2 = a 2 + b 2 − 2ab cos C
17 + 27 + 17
c = 402 + 552 − 2( 40)(55) cos 100° cm ∴ s = 2
m

= 73.4 cm (cor. to 1 d.p.) = 30.5 m


s − a = (30.5 − 17) m = 13.5 m
s − b = (30.5 − 27) m = 3.5 m
s − c = (30.5 − 17) m = 13.5 m

3
9 Basic Properties of CirclesMore about Trigonometry (II)

= s ( s − a )( s − b)( s − c ) 250° − 180° = 70°


∴ Area of △ABC = ∴ The required compass bearing is S70°W.
30.5 × 13.5 × 3.5 × 13.5 m 2
(d)
= 139 m 2 (cor. to 3 sig. fig.)

p. 192
1.

360° − 330° = 30°


∴ The required compass bearing is N30°W.

With the notations in the figure, 3. (a)


∠ABD = ∠BAC + ∠ACB
(a)
= 20° + 35°
= 55°
∴ The angle of elevation of A from B is 55°.
(b) ∠EAC = ∠ACB
= 35°
∠EAB = ∠EAC + ∠BAC
= 35° + 20° ∴ The required true bearing is 048°.
= 55° (b)
∴ The angle of depression of B from A is 55°.
(c) ∠EAC = 35°
∴ The angle of depression of C from A is 35°.

2. (a)
180° − 57° = 123
°
∴ The required true
bearing is 123°.
(c)

∴ The required compass bearing is N80°E.


(b)

180° + 62° = 242°


∴ The required true bearing is 242°.
(d)

180° − 120° = 60°


∴ The required compass bearing is S60°E.
(c)

360° − 28° = 332


°
∴ The required true
bearing is 332°.

4
Certificate Mathematics in Action Full Solutions 4B

4. By the sine formula,


CB 200 m
=
sin 35° sin 30°
200 sin 35°
CB = m
sin 30°
(alt. ∠s, AD // EC)
∠CBD = ∠ECB
= 65°
In △CBD,
CD
sin ∠CBD =
CB
CD = CB sin ∠CBD
200 sin 35°
=  × sin 65°  m
With the notations in the figure,  sin 30° 
= 207.9 m (cor. to 4 sig. fig.)
(a) ∠DBA = ∠BAF (alt. ∠s, DG // AF)
= 30° ∴ The height of the building is 207.9 m.
∴ The compass bearing of A from B is S30°E.
∠CBD = ∠CBA + ∠ABD p. 201
(b)
= 35° + 30° 1. (a) ∵∠HAD is the angle between planes ABGH
and ABCD.
= 65° ∴ The angle between planes ABGH and
∠GBC = 180° − ∠CBD
ABCD is 40°.
= 180° − 65° (b) ∵ ∠AHD is the angle between planes ABGH
= 115° and DCGH.
∴ The true bearing of C from B is 115°. ∠AHD + ∠HAD + ∠ADH = 180° (∠ sum of △)
(c) ∠BCH = ∠CBD (alt. ∠s, IH // DG) ∠AHD + 40° + 90° = 180°
= 65° ∠AHD = 50°
∴ The compass bearing of B from C is N65°W. ∴ The angle between planes ABGH and
(d) ∠ACI = ∠FAC (alt. ∠s, IH // AF) DCGH is 50°.
= 50° (c) ∵ ∠EAH is the angle between planes ABGH
∠HCA = 180° + ∠ACI and ABFE.
Reflex ∠EAH = ∠AHD (alt. ∠s, EA // HD)
= 180° + 50°
= 230° = 50°
∴ The true bearing of A from C is 230°. ∴ The angle between planes ABGH and
ABFE is 50°.
p. 195 (d) ∵ ∠EHA is the angle between planes ABGH
1. By the cosine formula, and EFGH.
QR 2 = PQ 2 + PR 2 − 2( PQ )( PR ) cos ∠QPR ∠EHA = ∠HAD
(alt. ∠s, EA // HD)
= 40°
QR = 202 + 152 − 2( 20)(15) cos 55° km
∴ The angle between planes ABGH and
= 17 km (cor. to the nearest km) EFGH is 40°.
∴ The distance between town Q and town R is 17 km. (e)

2.

With the notations in the figure,


∠ACB = ∠ECB − ∠ECA With the notations in the figure,
= 65° − 35° ∵ ∠EJA is the angle between planes ABGH
= 30° and EFCD.
JA = JD (property of rectangle)
∠CAB = ∠ECA (alt. ∠s, AD // EC)
∠JAD = ∠JDA (base ∠s, isos. △)
= 35°
= 40°
∠EJA = ∠JAD + ∠JDA (ext. ∠ of △)
= 40° + 40°
= 80°

5
9 Basic Properties of CirclesMore about Trigonometry (II)

∴ The angle between planes ABGH and Consider △BDH.


EFCD is 80°. HD
tan ∠HBD =
BD
2. (a) (i) ∵ VH is the height of the pyramid. 6
=
∴ H is the projection of V on plane 164
ABCD. ∠HBD = 25.1° (cor. to 3 sig. fig.)
∴ HA is the projection of VA on plane ∴ The angle between HB and BD is 25.1°.
ABCD.
∴ ∠VAH is the angle between VA and
2. (a)
plane ABCD.
(ii) ∵ VH is the height of the pyramid.
∴ H is the projection of V on plane
ABCD.
∴ HM is the projection of VM on plane
ABCD.
∴ ∠VMH is the angle between VM and
plane ABCD.
(b) (i) ∵ VM ⊥ AB and HM ⊥ AB
∴ ∠VMH is the angle between planes (b)
VAB and ABCD. The angle between the planes ACFD and
(ii) ∵ DH ⊥ VH and AH ⊥ VH BCFE is ∠ACB.
Consider △ABC.
∴ ∠DHA is the angle between planes
VHA and VHD. AC 2 + BC 2 − AB 2
cos ∠ACB =
2 AC ⋅ BC
p. 208 282 + 20 2 − 14 2
=
1. (a) (i) 2(28)( 20)
247
=
280
∠ACB = 28.1° (cor. to 3 sig. fig.)
∴ The angle between the planes ACFD and
BCFE is 28.1°.

3. (a) The angle between VA and plane ABCD is ∠


VAO.
(ii) AC = 212 + 202 cm
= 29 cm
1
∴ OA = AC = 14.5 cm (property of
2
rectangle)
Consider △VAO.
20
tan ∠VAO =
14.5
The angle between HB and plane HDCG is ∠VAO = 54.06° (cor. to 4 sig. fig.)
∠BHC.
∴ The angle between VA and plane ABCD is
CH = 6 2 + 10 2 m (Pyth. theorem) 54.06°.
= 136 m (b)
Consider △BCH.
8
tan ∠BHC =
136
∠BHC = 34.4° (cor. to 3 sig. fig.)
∴ The angle between HB and plane
HDCG is 34.4°.
(b) The angle between HB and BD is ∠HBD.
BD = 8 2 + 10 2 m (Pyth. theorem)
= 164 m
HD = GC = 6 m
Let M be the mid-point of AB.
∵ △VAB is an isosceles triangle.
∴ VM ⊥ AB

6
Certificate Mathematics in Action Full Solutions 4B

∵ △OAB is an isosceles triangle. (c)


∴ OM ⊥ AB
∴ AB is the line of intersection of the planes
VAB and ABCD.
∴ The angle between planes VAB and ABCD
is ∠VMO.
21
OM = cm
2
= 10.5 cm
Consider △VOM.
20 With the notations in the figure, consider
tan ∠VMO = △OAB.
10.5
∠VMO = 62.30° (cor. to 4 sig. fig.) AB
tan ∠AOB =
∴ The angle between the planes VAB and OA
ABCD is 62.30°. 300 m
=
(c) VO is the line of intersection of the planes VAO 200 m
and VDO. ∠AOB = 56.310°
∵ DO ⊥ VO and AO ⊥ VO ∠COB = ∠COA + ∠AOB
Reflex = 180° + 56.310°
∴ The angle between the planes VAO and
VDO is ∠DOA. = 236° (cor. to 3 sig. fig.)
Consider △DOA. ∴ The true bearing of B from O is 236°.
OD = OA (property of rectangle)
= 14.5 cm 2.
By the cosine formula,
OD 2 + OA2 − DA2
cos ∠DOA =
2OD ⋅ OA
14.52 + 14.52 − 212
=
2(14.5)(14.5)
41
= −
841
∠DOA = 92.79° (cor. to 4 sig. fig.)
∴ The angle between the planes VAO and
VDO is 92.79°. Let h m be the height of the tower TO.
Consider △TAO.
p. 216
hm
1. (a) Consider △TOA. tan 50° =
OA
TO
tan 30° = h
AO OA = m
tan 50°
TO = 200 tan 30° m
Consider △TBO.
= 115 m (cor. to 3 sig. fig.) hm
tan 30° =
∴ The height of the tower TO is 115 m. OB
(b) Consider △OAB. h
OB = m
∵ The man walks westwards to a point B. tan 30°
∴ B is due west of A. With the notations in the figure,
∴ ∠OAB = 90° ∠AOB = 90° − ∠BOC
OB 2 = OA2 + AB 2 = 90° − 40°
(Pyth. theorem)
OB = 2002 + 3002 m = 50°
Consider △OAB.
= 360.555 m
Consider △TOB. AB 2 = OA2 + OB 2 − 2(OA)(OB ) cos ∠AOB
2 2
TO h  h  h  h 
tan ∠TBO = 350 2 =   +   − 2   cos 50°
OB  tan 50°   tan 30°   tan 50°  tan 30° 
115.470 m 1 1 2 cos 50°  2
= 122 500 =  + − h
360.555 m  tan 2 50° tan 2 30° tan 30° tan 50° 
∠TBO = 17.8° (cor. to 3 sig. fig.) 122 500
h =
∴ The angle of elevation of T from B is 17.8 1 1 2 cos 50°
+ −
°. tan 2 50° tan 2 30° tan 30°tan 50°
= 258 (cor. to 3 sig. fig.)
∴ The height of the tower TO is 258 m.

7
9 Basic Properties of CirclesMore about Trigonometry (II)

Exercise Area of ∆ABC =


1
× AB × AC × sin ∠BAC
2
Exercise 9A (p. 169) 1
Level 1 32 = × 7 × 11 × sin ∠BAC
2
1
=  × 16 × 12 × sin 74°  cm 2 5. sin ∠BAC =
64
1. (a) Area of △ABC 2  77
= 92.3 cm 2 (cor. to 3 sig. fig.) ∠BAC = 56.22° or 180° − 56.22°
= 56.2° (cor. to 3 sig. fig.) or
1
=  × 10 × 8 × sin 142°  cm 2
(b) Area of △ABC 2  124° (cor. to 3 sig. fig.)
= 24.6 cm (cor. to 3 sig. fig.)
2

6. (a)
(c) BC = BA = 7 cm
1
=  × 7 × 7 × sin 43°  cm 2
Area of △ABC 2 
= 16.7 cm 2 (cor. to 3 sig. fig.)
(d) ∵ △ABC is an equilateral triangle.
∴ B = 60° and BC = AC = 4 cm
1 Join BD.
=  × 4 × 4 × sin 60°  cm 2
Area of △ABC  2  1
Area of △ABD =  × 5 × 7 × sin 68°  cm
2

= 6.93 cm (cor. to 3 sig. fig.)


2 2 
∴ Area of ABCD
= 2 × area of △ABD
1
ABC = × AC × BC × sin C 1
=  2 × × 5 × 7 × sin 68°  cm
2
2
2. (a) Area of △  2 
1
7.5 = × x × 5 × sin 65° 2
2 = 32.5 cm (cor. to 3 sig. fig.)
x = 3.31 (cor. to 3 sig. fig.) (b)
1
ABC = × AB × AC × sin A
2
(b) Area of △ 1
29 = × 12 × x × sin 138°
2
x = 7.22 (cor. to 3 sig. fig.) Join AC.
∠ADC = ∠ABC (opp. ∠s of // gram)
1 = 132°
PQR = × PR × QR × sin R
2 1
Area of △ACD =  × 8 × 6 × sin 132°  cm
2
1 2 
3. (a) Area of △ 15 = × 8 × 5 × sin θ
2 ∴ Area of ABCD
θ = 48.6° (cor. to 3 sig. fig.) or = 2 × area of △ACD
131° (rejected) 1
=  2 × × 8 × 6 × sin 132°  cm
2

1  2 
PQR = × PQ × PR × sin P 2
2 = 35.7 cm (cor. to 3 sig. fig.)
1
(b) Area of △ 20 = × 7.4 × 12.3 × sin θ
2
7.
θ = 26.1° (cor. to 3 sig. fig.) or
154° (rejected)

1
ABC = × AB × AC × sin ∠BAC
4. Area of △ 2 Join BD.
1 AB = AD (property of rhombus)
55 = × 12 × 10 × sin ∠BAC
2
= 14 cm
11
sin ∠BAC = 1
Area of △ABD =  × 14 × 14 × sin 48°  cm
2
12
2 
∠BAC = 66.4° (rejected) or
∴ Area of rhombus ABCD
114° (cor. to 3 sig. fig.)
= 2 × area of △ABD
1
=  2 × × 14 × 14 × sin 48°  cm
2
 2 
2
= 146 cm (cor. to 3 sig. fig.)

8
Certificate Mathematics in Action Full Solutions 4B

8. Area of the shaded segment


= area of sector OAE − area of △OAE
72° 1
=  π × 82 × − × 8 × 8 × sin 72°  cm 2
 360° 2 
Join AC. = 9.78 cm 2 (cor. to 3 sig. fig.)
∠CBA + ∠DAB = 180° (int. ∠s, CB // DA)
∠CBA + 160° = 180° 12. Let AB = x cm and BC = (x + 2) cm, where x is an
∠CBA = 20° integer.
Area of parallelogram ABCD ∵ 10 cm2 < area of △ABC < 20 cm2
= 2 × area of ABC 1
10 cm 2 < × AB × AC × sin ∠ABC < 20 cm 2
1 2
= 2 × × AB × BC × sin ∠CBA 1
2 ∴ 10 < x ( x + 2) sin 30° < 20
2
1
38 cm 2 = 2 × × 14 cm × BC × sin 20° 1
2 10 < x ( x + 2) < 20
∴ 4
38
BC = cm 40 < x ( x + 2) < 80
14 sin 20°
= 7.94 cm (cor. to 3 sig. fig.) ∴ x must satisfy the inequality 40 < x(x + 2) < 80.
Take x = 6,
x ( x + 2) = 6(6 + 2)
9. ∵ △ABC is an isosceles triangle.
= 48
∴ AC = AB ∴ The possible lengths of AB and BC are 6 cm and
= 6.5 cm 8 cm respectively. (or any other reasonable
∠ACB = ∠ABC △ answers)
(base ∠s, isos. )
= 28°
13. ∵ The areas of △ABC and △XYZ are the same.
∠BAC + ∠ABC + ∠ACB = 180° (∠ sum of △)

∠BAC + 28° + 28° = 180°
1 1
∠BAC = 124° × AB × BC × sin ∠ABC = × XY × YZ × sin ∠XYZ
2 2
1 1 1
= × AB × AC × sin ∠BAC × 3 × 4 × sin ∠ABC = × 6 × 2 × sin ∠XYZ
2 2 2
Area of the triangle  1
=  × 6.5 × 6.5 × sin 124°  cm 2 sin ∠ABC = sin ∠XYZ
2 
∠ABC = ∠XYZ (rejected) or ∠ABC = 180° − ∠XYZ
= 17.5 cm 2 (cor. to 3 sig. fig.) Take ∠ABC = 30°,
30° = 180° − ∠XYZ
10. Area of quadrilateral ABCD ∠XYZ = 150°
= area of △ACD + area of △ABC ∴ The possible values of ∠ABC and ∠XYZ are 30°
1 and 150° respectively. (or any other reasonable
× AD × AC × sin ∠CAD +
= 2 answers)
1
× AC × AB × sin ∠CAB Level 2
2
1 1 ∠ADB = ∠DBC + ∠BCD
=  × 5 × 7 × sin 28° + × 7 × 10 × sin 52°  cm (ext. ∠ of △)
2
14.
2 2  = 35° + 50°
2 = 85°
= 35.8 cm (cor. to 3 sig. fig.)
Area of △ABC
= area of △BCD + area of △ABD
11. (a) Area of the shaded segment 1
= area of sector OAB − area of △OAB = × BC × BD × sin ∠DBC +
2
135° 1 1
=  π × 202 × − × 20 × 20 × sin 135°  cm 2 × AD × BD × sin ∠ADB
 360° 2  2
= 330 cm 2 (cor. to 3 sig. fig.)  1 × 7 × 5.38 × sin 35° + 
  2
(b) ∵ EA = AB = BC = CD = DE (given) = 2  cm
 1 × 3 × 5.38 × sin 85° 
∠EOA = ∠AOB = ∠BOC 2 
= ∠COD = ∠DOE (equal chords, equal ∠s) = 18.8 cm 2 (cor. to 3 sig. fig.)

360°
∠EOA =
5 15. ∵ △ABC is an isosceles triangle.
= 72° ∴ AC = BC = 9 cm
∠BAC = ∠ABC △
(base ∠s, isos. )
= 70°

9
9 Basic Properties of CirclesMore about Trigonometry (II)

∠ACB + ∠BAC + ∠ABC = 180° (∠ sum of △) Area of major segment BAC


∠ACB + 70° + 70° = 180° = area of sector OACB + area of △OAB
230° 1
∠ACB = 40° =  π × 12 2 × + × 12 × 12 × sin 130°  cm 2
 360° 2 
∠DAC = ∠ACB (alt. ∠s, AD // BC)
= 344 cm 2 (cor. to 3 sig. fig.)
= 40°
Area of trapezium ABCD
= area of △ABC + area of △ACD OC = CA
1 19. 8 (given)
= × AC × BC × sin ∠ACB + = cm
2 2
1 = 4 cm
× AD × AC × sin ∠DAC
2 Area of the shaded region
1 1
=  × 9 × 9 × sin 40° + × 6 × 9 × sin 40°  cm 2 = area of sector OAB − area of △OCB
2 2  45° 1

=  π × 82 × − × 4 × 8 × sin 45°  cm 2
= 43. 4 cm 2 (cor. to 3 sig. fig.)  360° 2 
= 13.8 cm 2 (cor. to 3 sig. fig.)
16. Consider △ABD.
BD 2 = AB 2 + AD 2 (Pyth. theorem) 20.
BD = 52 + 12 2 cm
= 13 cm
∵ Area of △ABD = area of △BCD
1 1
× AB × AD = × BD × BC × sin ∠DBC
2 2
1 1
× 5 × 12 = × 13 × 8 × sin ∠DBC Join OC.
2 2
15 ∵ OC = OA (radii)
sin ∠DBC = ∠ ACO = ∠ CAB △
26 ∴ (base ∠s, isos. )
∠DBC = 35.23° or 180° − 35.23° = 35°
= 35.2° (cor. to 3 sig. fig.) or ∠BOC = ∠ACO + ∠CAB (ext. ∠ of △)
= 35° + 35°
145° (cor. to 3 sig. fig.) (rejected)
= 70°
Area of the shaded region
KC = AK
= area of sector OBC − area of △OBC
1 70° 1
= AC 
= π × 72 × − × 7 × 7 × sin 70°  cm 2
17. 2 (diags. of // gram)  360° 2 
 1 
=  × 10  cm = 6.91 cm (cor. to 3 sig. fig.)
2
2 
= 5 cm
KD = BK 21. (a) Consider △ABC and △ADE.
1 ∠BAC = ∠DAE = 70° (common angle)
= BD (diags. of // gram) ∠ABC = ∠ADE (corr. ∠s, BC // DE)
2
1 ∠ACB = ∠AED (corr. ∠s, BC // DE)
=  × 14  cm ∴ △ABC ~ △ADE (AAA)
2 
= 7 cm AB AC
= (corr. sides, ~ s)
Area of parallelogram ABCD AD AE
= 4 × area of △CDK AB AC
=
1 AB + BD AC + CE
= 4 × × KC × KD × sin ∠DKC 6 cm 4
2 =
 4 × 1 × 5 × 7 × sin 70°  cm 2 6 cm + BD 4+2
= 6 cm + BD = 9 cm
 2 
= 65.8 cm 2 (cor. to 3 sig. fig.) BD = 3 cm
(b) Area of BDEC
= area of △ADE − area of △ABC
18. Reflex ∠AOB = 360° − 130° (∠s at a pt.)
= 230°  1 × (6 + 3) × ( 4 + 2) × sin 70° −
  2
= 2  cm
1
 × 6 × 4 × sin 70° 
2 
= 14.1 cm 2 (cor. to 3 sig. fig.)

10
Certificate Mathematics in Action Full Solutions 4B

OP = OQ 4. By the sine formula,


22. (a) = BC (radii) AC AB
=
= 10 cm sin B sin C
9 cm 10 cm
=
OD = OC (given) sin x sin 48°
1 9 sin 48°
= CD sin x =
(b) 2 10
1 x = 42.0° (cor. to 3 sig. fig.) or
=  × 10  cm

2  138° (cor. to 3 sig. fig.) (rejected)
= 5 cm
Consider △POD. 5. By the sine formula,
OD a b
cos ∠POD = =
OP sin A sin B
5 6 cm 12 cm
= =
10 sin 60° sin B
∠POD = 60° 12 sin 60°
sin B =
(c) By similar argument, ∠ROC = 60°. 6
∠POD + ∠POR + ∠ROC = 180° (adj. ∠s on st. line) = 1.73 (cor. to 3 sig. fig.)
60° + ∠POR + 60° = 180° ∵ sin B ≤ 1 for all values of B.
∠POR = 60° ∴ There are no solutions for B.
Area of the shaded region
= area of sector OPR − area of △OPR 6. By the sine formula,
a b
60° 1 =
=  π × 10 2 × − × 10 × 10 × sin 60°  cm 2 sin A sin B
 360° 2 
3 cm 2 cm
= 9.06 cm 2 (cor. to 3 sig. fig.) =
sin 60° sin B
2 sin 60°
sin B =
Exercise 9B (p. 178) 3
Level 1 =1
1. By the sine formula, B = 90°
BC AC
=
sin A sin B
x cm 4 cm 7. By the sine formula,
= a b
sin 75° sin 44° =
4 sin 75° sin A sin B
x = 12 cm 6 cm
sin 44° =
= 5.56 (cor. to 3 sig. fig.) sin 60° sin B
6 sin 60°
sin B =
12
2. By the sine formula, B = 25.7° (cor. to 3 sig. fig.) or
AC BC
=
sin B sin A 154° (cor. to 3 sig. fig.) (rejected)
x cm 7 cm
=
sin 116° sin 28° 8. By the sine formula,
7 sin 116° a b
x = =
sin 28° sin A sin B
= 13.4 (cor. to 3 sig. fig.) 6 cm 6 cm
=
sin 70° sin B
6 sin 70°
3. By the sine formula, sin B =
6
AC AB
= B = 70° or 110° (rejected)
sin B sin C
14.7 cm 12 cm
=
sin 68° sin x A + B + C = 180°
12 sin 68° 9. 57° + B + 81° = 180° (∠ sum of △)
sin x =
14.7 B = 42°
x = 49.2° (cor. to 3 sig. fig.) or
131° (cor. to 3 sig. fig.) (rejected)

11
9 Basic Properties of CirclesMore about Trigonometry (II)

By the sine formula, c b


=
a b sin C sin B
=
sin A sin B 8 cm 5.5 cm
=
a 8 cm sin C sin 36°
=
sin 57° sin 42° 8 sin 36°
sin C =
8 sin 57° 5.5
a = cm
sin 42° C = 58.8° (cor. to 3 sig. fig.) or 121° (cor. to 3 sig. fig.)
= 10.0 cm (cor. to 3 sig. fig.)
When C = 58.8°,
By the sine formula, A + B + C = 180°(∠ sum of △)
c b A + 36° + 58.755° = 180°
=
sin C sin B
A = 85.2° (cor. to 3 sig. fig.)
c 8 cm
=
sin 81° sin 42° By the sine formula,
8 sin 81° a b
c = cm =
sin 42° sin A sin B
= 11.8 cm (cor. to 3 sig. fig.) a 5.5 cm
=
sin 85.245° sin 36°
5.5 sin 85.245°
A + B + C = 180° a = cm
sin 36°
10. A + 110° + 26° = 180° (∠ sum of △)
= 9.32 cm (cor. to 3 sig. fig.)
A = 44°
When C = 121°,
By the sine formula, A + B + C = 180°(∠ sum of △)
b a A + 36° + 121.245° = 180°
=
sin B sin A
A = 22.8° (cor. to 3 sig. fig.)
b 6 cm
=
sin 110° sin 44° By the sine formula,
6 sin 110° a b
b = cm =
sin 44° sin A sin B
= 8.12 cm (cor. to 3 sig. fig.) a 5.5 cm
=
sin 22.755° sin 36°
By the sine formula, 5.5 sin 22.755°
c a a = cm
= sin 36°
sin C sin A
= 3.62 cm (cor. to 3 sig. fig.)
c 6 cm
=
sin 26° sin 44°
6 sin 26° 13. By the sine formula,
c = cm b c
sin 44° =
= 3.79 cm (cor. to 3 sig. fig.) sin B sin C
b c
=
sin 45° sin 30°
A + B + C = 180° c sin 45°
b =
11. 125° + 32° + C = 180° (∠ sum of △) sin 30°
C = 23° b = 2c
By the sine formula, Take b = 2 ,
a b 2 = 2c
=
sin A sin B c =1
a 10 cm ∴ The possible values of b and c are 2 and 1
=
sin 125° sin 32° respectively. (or any other reasonable answers)
10 sin 125°
a = cm
sin 32° 14. Take B = 90°, and by the sine formula,
= 15.5 cm (cor. to 3 sig. fig.) b a
=
By the sine formula, sin B sin A
c b b 6 cm
= =
sin C sin B sin 90° sin 30°
c 10 cm 6 sin 90°
= b = cm
sin 23° sin 32° sin 30°
10 sin 23° = 12 cm
c = cm A + B + C = 180° (∠ sum of △)
sin 32°
= 7.37 cm (cor. to 3 sig. fig.) 30° + 90° + C = 180°
C = 60°
12. By the sine formula,

12
Certificate Mathematics in Action Full Solutions 4B

By the sine formula, ∠BAM + ∠AMB + ∠ABM = 180° (∠ sum of △)


c a 30° + ∠AMB + 45° = 180°
=
sin C sin A ∠AMB = 105°
c 6 cm By the sine formula,
=
sin 60° sin 30° AB BM
6 sin 60° =
c = cm sin ∠AMB sin ∠BAM
sin 30° x cm 6 cm
= 6 3 cm =
sin 105° sin 30°
∴ The possible values for b, c, B and C are 12 cm, 6 sin 105°
x =
6 3 cm , 90° and 60° respectively. (or any other sin 30°
reasonable answers) = 11.6 (cor. to 3 sig. fig.)
AM BM
Level 2 =
sin ∠ABM sin ∠BAM
∠ABC + ∠BAC = ∠ACD
15. (ext. ∠ of △) y cm
=
6 cm
30° + ∠BAC = 50°
sin 45° sin 30°
∠BAC = 20° 6 sin 45°
Consider △ACD. y =
sin 30°
AD = 8.49 (cor. to 3 sig. fig.)
sin 50° =
AC
x ∠AMC = ∠BAM + ∠ABM (ext. ∠ of △)
AC = cm
sin 50° = 30° + 45°
By the sine formula, = 75°
AC BC In △AMC,
=
sin ∠ABC sin ∠BAC ∠MAC + ∠AMC + ∠ACM = 180° (∠ sum of △)
x 30° + 75° + ∠ACM = 180°
cm 40 cm
sin 50° =
sin 30° sin 20° ∠ACM = 75°
40 sin 30° sin 50°
x =
sin 20° ∵ ∠ACM = ∠AMC
= 44.8 (cor. to 3 sig. fig.) AC = AM (sides opp.
equal ∠s)
∠ABD + ∠BAD + ∠ADB = 180° z=y

16. (∠ sum of △) = 8.49 (cor. to 3 sig. fig.)
∠ABD + 100° + 30° = 180°
∠ABD = 50°
∠DBC = ∠ADB (alt. ∠s, BC // AD) 18. In △ACD,
= 30° AC = AD (given)

∠DBC + ∠BDC + ∠BCD = 180° ∠ACD = ∠ADC (base ∠s, isos. )
(∠ sum of △)
30° + 80° + ∠BCD = 180° = 65°
∠BCD = 70° ∠CAD + ∠ACD + ∠ADC = 180° (∠ sum of )
By the sine formula, ∠CAD + 65° + 65° = 180°
BD AD ∠CAD = 50°
=
sin ∠BAD sin ∠ABD By the sine formula,
BD 18 cm CD AC
= =
sin 100° sin 50° sin ∠CAD sin ∠ADC
18 sin 100° x cm 7 cm
BD = cm =
sin 50° sin 50° sin 65°
By the sine formula, 7 sin 50°
BC BD x =
= sin 65°
sin ∠BDC sin ∠BCD = 5.92 (cor. to 3 sig. fig.)
18 sin 100°
x cm cm In △ABC,
= sin 50°
sin 80° sin 70° ∠ABC + ∠BAC = ∠ACD (ext. ∠ of △)
18 sin 100° sin 80° 40° + ∠BAC = 65°
x =
sin 50° sin 70° ∠BAC = 25°
= 24.3 (cor. to 3 sig. fig.) ∠ABC + ∠BAC + ∠ACB = 180° (∠ sum of △)
40° + 25° + ∠ACB = 180°
17. In △ABM, ∠ACB = 115°

13
9 Basic Properties of CirclesMore about Trigonometry (II)

By the sine formula, 22. By the sine formula,


BC AC a b
= =
sin ∠BAC sin ∠ABC sin A sin B
y cm 7 cm 12 cm 15 cm
= =
sin 25° sin 40° sin 45° sin B
7 sin 25° 15 sin 45°
y = sin B =
sin 40° 12
= 4.60 (cor. to 3 sig. fig.) 5 2
=
8
AB AC
= B = 62 .1° (cor. to 3 sig. fig.) or
sin ∠ACB sin ∠ABC ∴
z cm 7 cm 118° (cor. to 3 sig. fig.)
=
sin 115° sin 40°
7 sin 115°
z = 23. By the sine formula,
sin 40°
b c
= 9.87 (cor. to 3 sig. fig.) =
sin B sin C
6 cm 7 cm
=
19. By the sine formula, sin B sin 50°
b a 6 sin 50°
= sin B =
sin B sin A 7
12 cm 8 cm B = 41.0° (cor. to 3 sig. fig.) or
=
sin B sin 150°
139° (cor. to 3 sig. fig.) (rejected)
12 sin 150°
sin B = A + B + C = 180° (∠ sum of △)
8
3 A + 41.042° + 50° = 180°
=
4 A = 89.0° (cor. to 3 sig. fig.)
B = 48.6° (cor. to 3 sig. fig.) (rejected) or
By the sine formula,
131° (cor. to 3 sig. fig.) (rejected) a c
∴ There are no solutions for B. =
sin A sin C
a 7 cm
20. By the sine formula, =
sin 88.958° sin 50°
b a 7 sin 88.958°
=
sin B sin A a = cm
sin 50°
6 cm 3.5 cm = 9.14 cm (cor. to 3 sig. fig.)
=
sin B sin 30°
6 sin 30°
sin B =
3.5 24. By the sine formula,
6 a b
= =
7 sin A sin B
B = 59.0° (cor. to 3 sig. fig.) or 14 cm 8 cm
∴ =
sin A sin 62°
121° (cor. to 3 sig. fig.) 14 sin 62°
sin A =
8
21. By the sine formula, = 1.55 (cor. to 3 sig. fig.)
b a ∵ sin A ≤ 1 for all values of A.
=
sin B sin A ∴ There are no solutions for A.
12 cm 9 cm ∴ No triangle can be formed.
=
sin B sin 70°
12 sin 70°
sin B =
9 (cor. to 3 sig. fig.)
= 1.25 (cor. to 3 sig. fig.) sin A ?1 for all values of A.
∵ sin B ≤ 1 for all values of B. \ There are no solutions for A.
∴ There are no solutions for B. \ No triangle can be formed.
25. By the sine formula,
b c
=
sin B sin C
(cor. to 3 sig. fig.) 6.5 cm 9 cm
=
Q sin B ?1 for all values of B. sin B sin 55°
\ There are no solutions for B. 6.5 sin 55°
sin B =
9

14
Certificate Mathematics in Action Full Solutions 4B

∴ B = 36.3° (cor. to 3 sig. fig.) or A + B + C = 180°(∠ sum of △)


144° (cor. to 3 sig. fig.) (rejected) 43° + B + 114.587° = 180°
A + B + C = 180° (∠ sum of △) B = 22.4° (cor. to 3 sig. fig.)
A + 36.271° + 55° = 180° By the sine formula,
A = 88.7° (cor. to 3 sig. fig.) b a
=
sin B sin A
b 7.5 cm
=
sin 22.413° sin 43°
7.5 sin 22.413°
\ (cor. to 3 sig. fig.) or b = cm
144?(cor. to 3 sig. fig.) (rejected) sin 43°
(?sum of △) = 4.19 cm (cor. to 3 sig. fig.)

(cor. to 3 sig. fig.)


By the sine formula, (cor. to 3 sig. fig.)
a c
=
sin A sin C ∠CAB + ∠ACB + ∠CBA = 180°(∠ sum of △)
a 9 cm 27.
= 25° + ∠ACB + 48° = 180°
sin 88.729° sin 55° (?sum of △)
9 sin 88.729° ∠ACB = 107°
a = cm
sin 55°
= 11.0 cm (cor. to 3 sig. fig.)
By the sine formula,
BC AB
=
sin ∠CAB sin ∠ACB
(cor. to 3 sig. fig.) BC 10 cm
26. By the sine formula, =
sin 25° sin 107°
c a 10 sin 25°
= BC = cm
sin C sin A sin 107°
10 cm 7.5 cm ∠BCT = ∠CAB (∠
=
sin C sin 43°
= 25°
10 sin 43°
sin C =
7.5
C = 65.4° (cor. to 3 sig. fig.) or
115° (cor. to 3 sig. fig.)
(?in alt. segment)
∠BCT + ∠CTB = ∠CBA
25° + ∠CTB = 48°
\ (cor. to 3 sig. fig.) or
∠CTB = 23°
(cor. to 3 sig. fig.)
When C = 65.4°,
A + B + C = 180° (∠ sum of △)
(ext. ∠ ?of △)
43° + B + 65.413° = 180° ?
B = 71.6° (cor. to 3 sig. fig.) ∠CBA + ∠CBT = 180°
(?sum of △) 48° + ∠CBT = 180°
∠CBT = 132°
(cor. to 3 sig. fig.)
By the sine formula,
b a (adj. ∠s 苫 on
=
sin B sin A
b 7.5 cm st. line)
=
sin 71.587° sin 43°
7.5 sin 71.587°
b = cm
sin 43°
= 10.4 cm (cor. to 3 sig. fig.) By the sine formula,

When C = 115°,

15
9 Basic Properties of CirclesMore about Trigonometry (II)

CT BC
=
sin ∠CBT sin ∠CTB
10 sin 25°
CT cm
= sin 107°
sin 132° sin 23°
10 sin 25° sin 132°
CT = cm
sin 23° sin 107°
= 8.41 cm (cor. to 3 sig. fig.)

(cor. to 3 sig. fig.)


28. ∵ ABGF and CDEG are two squares.
∴ ∠BGF = 90°, ∠CGE = 90°, BG = FG and
CG = EG = 2 cm
Consider △BCG.
∠CBG + ∠BGC + ∠BCG = 180° (∠ sum of △)
∠CBG + 105° + 30° = 180°
∠CBG = 45°
By the sine formula,
BG CG
=
sin ∠BCG sin ∠CBG
BG 2 cm
=
sin 30° sin 45°
2 sin 30°
BG = cm
sin 45°
= 1 cm
∠BGC + ∠BGF + ∠FGE + ∠CGE = 360° (∠s at
105° + 90° + ∠FGE + 90° = 360° a pt.)
∠FGE = 75°
Area of the hexagon ABCDEF
= area of △BCG + area of ABGF + area of △EFG +
area of CDEG
1
= × BG × CG × sin ∠BGC + BG 2 +
2
1
× FG × EG × sin ∠FGE + CG 2
2
 1 × 1 × 2 × sin 105° + 12 + 
  2
= 2  cm
1
 × 1 × 2 × sin 75° + ( 2 ) 2 
2
= 4.37 cm 2 (cor. to 3 sig. fig.)

Exercise 9C (p. 185)


Level 1
1. By the cosine formula,
b 2 = a 2 + c 2 − 2ac cos B
b = 10 2 + 8 2 − 2(10)(8) cos 40° cm

x = 10 2 + 8 2 − 2(10)(8) cos 40°


= 6.44 (cor. to 3 sig. fig.)

16
Certificate Mathematics in Action Full Solutions 4B

2. By the cosine formula, a2 + c2 − b2


cos B =
a 2 = b 2 + c 2 − 2bc cos A 2ac
a = 14 2 + 9 2 − 2(14)(9) cos 145° cm 14.106 2 + 12 2 − 17 2
∴ =
2(14.106)(12)
x = 14 + 9 − 2(14)(9) cos 145°
2 2
B = 80.8° (cor. to 3 sig. fig.)
= 22.0 (cor. to 3 sig. fig.)
A + B + C = 180° (∠ sum of △)
55° + 80.825° + C = 180°
3. By the cosine formula,
C = 44.2° (cor. to 3 sig. fig.)
b2 + c2 − a2
cos A =
2bc
8. By the cosine formula,
4.5 2 + 7 2 − 6 2
cosθ = b 2 = a 2 + c 2 − 2ac cos B
2( 4.5)(7)
θ = 58.1° (cor. to 3 sig. fig.) b = 112 + 8 2 − 2(11)(8) cos 21° cm
= 4.55 cm (cor. to 3 sig. fig.)
4. By the cosine formula,
b2 + c2 − a2
a2 + c2 − b2 cos A =
cos B = 2bc
2ac
4.5486 2 + 8 2 − 112
9 2 + 6 2 − 112 =
cosθ = 2( 4.5486)(8)
2(9)(6)
∴ A = 120° (cor. to 3 sig. fig.)
1
=− A + B + C = 180° (∠ sum of △)
27
θ = 92.1° (cor. to 3 sig. fig.) 119.93° + 21° + C = 180°
C = 39.1° (cor. to 3 sig. fig.)
5. The largest angle of △ABC is C.
By the cosine formula, 9. By the cosine formula,
a2 + b2 − c2 b2 + c2 − a2
cos C = cos A =
2ab 2bc
8 2 + 8.5 2 − 9 2 12 2 + 10 2 − 15 2
= =
2(8)(8.5) 2(12)(10)
13 19
= =
32 240
C = 66.0° (cor. to 3 sig. fig.) A = 85.5° (cor. to 3 sig. fig.)
∴ The largest angle of △ABC is 66.0°. a2 + c2 − b2
cos B =
2ac
∠BAD + ∠ABC = 180°
6. (int. ∠s, AD // BC) 15 2 + 10 2 − 12 2
∠BAD + 60° = 180° =
2(15)(10)
∠BAD = 120° 181
AD = BC = 12 cm (opp. sides of // gram) =
300
Consider △ABC.
B = 52.9° (cor. to 3 sig. fig.)
AC 2 = AB 2 + BC 2 − 2( AB)( BC ) cos ∠ABC
A + B + C = 180° (∠ sum of △)
AC = 8 2 + 12 2 − 2(8)(12) cos 60° cm 85.4593° + 52.8910° + C = 180°
= 10.6 cm (cor. to 3 sig. fig.) C = 41.6° (cor. to 3 sig. fig.)
Consider △ABD.
BD 2 = AB 2 + AD 2 − 2( AB)( AD) cos ∠BAD 10. By the cosine formula,
b2 + c2 − a2
BD = 8 2 + 12 2 − 2(8)(12) cos 120° cm cos A =
2bc
= 17.4 cm (cor. to 3 sig. fig.)
10 + 5 2 − 7.5 2
2
=
2(10)(5)
7. By the cosine formula,
11
a 2 = b 2 + c 2 − 2bc cos A =
16
a = 17 2 + 12 2 − 2(17)(12) cos 55° cm A = 46.6° (cor. to 3 sig. fig.)
= 14.1 cm (cor. to 3 sig. fig.)

17
9 Basic Properties of CirclesMore about Trigonometry (II)

a 2 + c2 − b2 14.
cos B =
2ac
7.5 + 52 − 10 2
2
=
2(7.5)(5)
1
=−
4
B = 104° (cor. to 3 sig. fig.)
A + B + C = 180° (∠ sum of △)
46.567° + 104.478° + C = 180° Join AC.
By the cosine formula,
C = 29.0° (cor. to 3 sig. fig.)
AC 2 = BC 2 + AB 2 − 2( BC )( AB) cos ∠ABC

Level 2 AC = 12 2 + 132 − 2(12)(13) cos 75° cm


11. By the cosine formula, = 15.240 cm
AC 2 = AB 2 + BC 2 − 2( AB)( BC ) cos ∠ABC By the cosine formula,
CD 2 + AD 2 − AC 2
AC = 4 2 + 6 2 − 2(4)(6) cos 100° cm cos ∠ADC =
2(CD )( AD )
x = 7.77 (cor. to 3 sig. fig.)
112 + 10 2 − 15.240 2
By the cosine formula, cos θ =
2(11)(10)
AD 2 = AC 2 + DC 2 − 2( AC )( DC ) cos ∠ACD
θ = 92.9° (cor. to 3 sig. fig.)
AD = 7.7676 2 + 7 2 − 2(7.7676)(7) cos 42° cm
y = 5.34 (cor. to 3 sig. fig.) 15. Consider △ABD.
By the cosine formula,
12. By the cosine formula, BD 2 + AD 2 − AB 2
cos ∠ADB =
AD 2 = AB 2 + BD 2 − 2( AB )( BD) cos ∠ABD 2( BD)( AD)
AD = 3 2 + 6 2 − 2(3)(6) cos 48° cm (4 + 5) 2 + 82 − 6 2
=
= 4.57 cm (cor. to 3 sig. fig.) 2( 4 + 5)(8)
109
∵ ABCD is a parallelogram. =
∴ BC = AD and DC = AB = 3 cm 144
BC = AD Consider △ACD.
By the cosine formula,
x = 4.57 (cor. to 3 sig. fig.)
AC 2 = CD 2 + AD 2 − 2(CD )( AD) cos ∠ADC
By the cosine formula,
BC 2 + BD 2 − DC 2  109 
cos ∠CBD = AC = 5 2 + 8 2 − 2(5)(8)  cm
2( BC )( BD)  144 
4.5729 2 + 6 2 − 32 x = 5.33 (cor. to 3 sig. fig.)
cos θ =
2(4.5729)(6)
θ = 29.2° (cor. to 3 sig. fig.) 16. Consider △ABD.
By the cosine formula,
AD 2 = BD 2 + AB 2 − 2( BD)( AB) cos ∠ABD
13. By the cosine formula,
AC 2 + BC 2 − AB 2 AD = (7 + 5) 2 + 10 2 − 2(7 + 5)(10) cos 36° cm
cos ∠ACB =
2( AC )( BC ) x = 7.06 (cor. to 3 sig. fig.)
4 +6 −5
2 2 2
Consider △ABC.
=
2(4)(6) By the cosine formula,
9 AC 2 = BC 2 + AB 2 − 2( BC )( AB ) cos ∠ABC
=
16 AC = 7 2 + 10 2 − 2(7)(10) cos 36° cm
∠ECD = ∠ACB
∠ = 5.9781 cm
cos ∠ECD = cos ∠ACB (vert. opp. s)
Consider △ACD.
9 By the cosine formula,
=
16 AD 2 + AC 2 − CD 2
By the cosine formula, cos ∠CAD =
2( AD)( AC )
ED 2 = CE 2 + CD 2 − 2(CE )(CD ) cos ∠ECD
7.0595 2 + 5.97812 − 5 2
cos θ =
 9 2(7.0595)(5.9781)
ED = 7 2 + 8 2 − 2(7)(8)  cm
 16  θ = 44.1° (cor. to 3 sig. fig.)
x = 7.07 (cor. to 3 sig. fig.)

18
Certificate Mathematics in Action Full Solutions 4B

17. (a) Consider △ABC. 19. ∵ b : c = 3 : 2


By the cosine formula, b 3
=
BC 2 + AC 2 − AB 2 ∴ c 2
cos C =
2( BC )( AC ) 3
b= c
(5 + 6) 2 + (7 + 3) 2 − 9 2 2
= By the cosine formula,
2(5 + 6)(7 + 3)
a 2 = b 2 + c 2 − 2bc cos A
7
= 2
11 3  3 
14 2 cm 2 =  c  + c 2 − 2 c (c) cos 60°
7 2
  2 
∴ cos C =
11 9 3
196 cm 2 = c 2 + c 2 − c 2
(b) Consider △CMN. 4 2
By the cosine formula, 7 2
MN 2 = NC 2 + MC 2 − 2( NC )( MC ) cos C 196 cm = c
2

4
7 784
MN = 6 2 + 3 2 − 2(6)(3)  cm c= cm
 11  7
= 4.70 cm (cor. to 3 sig. fig.) 28
= cm
7
18. (a) Consider △ABC. = 4 7 cm
By the cosine formula,
3
AC 2 = BC 2 + AB 2 − 2( BC )( AB) cos ∠ABC b= c
2
AC = 8 2 + 112 − 2(8)(11) cos 70° cm 3 
=  × 4 7  cm
= 11.2 cm (cor. to 3 sig. fig.)  2 
∵ ABCD is a parallelogram. = 6 7 cm
∴ DC = AB = 11 cm
∠BCD + ∠ABC = 180° (int. ∠s, AB // DC) 20. (a) Consider △ABC.
∠BCD + 70° = 180° By the cosine formula,
∠BCD = 110° AC 2 = BC 2 + AB 2 − 2( BC )( AB) cos ∠ABC
Consider △BCD.
= [32 + 4 2 − 2(3)(4) cos θ ] cm 2
BD 2 = DC 2 + BC 2 − 2( DC )( BC ) cos ∠BCD
= ( 25 − 24 cos θ ) cm 2
BD = 11 + 8 − 2(11)(8) cos 110° cm
2 2
Consider △ADC.
= 15.7 cm (cor. to 3 sig. fig.) ∠ABC + ∠ADC = 180° (opp. ∠s, cyclic quad.)
(b) ∵ ABCD is a parallelogram. θ + ∠ADC = 180°
BK = KD ∠ADC = 180° − θ
1 By the cosine formula,
= BD
∴ 2 AC 2 = DC 2 + AD 2 − 2( DC )( AD) cos ∠ADC
1  = [6 2 + 5 2 − 2(6)(5) cos (180° − θ )] cm 2
=  × 15.659  cm
2  = (61 + 60 cos θ ) cm 2
= 7.830 cm 25 − 24 cos θ = 61 + 60 cos θ
CK = KA − 36 = 84 cos θ

1 3
= AC cos θ = −
2 7
1 
=  ×11.172  cm  3
2  AC = 25 − 24 −  cm
 7
= 5.586 cm (b)
Consider △BCK. 247
= cm
By the cosine formula, 7
CK 2 + BK 2 − BC 2 = 5.94 cm (cor. to 3 sig. fig.)
cos ∠BKC =
2(CK )( BK )
5.586 2 + 7.830 2 − 8 2
=
2(5.586)(7.830)
∠BKC = 71.0° (cor. to 3 sig. fig.)
∴ The acute angle between the diagonals is
71.0°.

19
9 Basic Properties of CirclesMore about Trigonometry (II)

Exercise 9D (p. 189) a+b+c


5. Let s = , where a = 25 m, b = 23 m and
Level 1 2
1. a = b = c = 6 cm c = 18 m.
a+b+c 25 + 23 + 18
Let s =
2
. ∴ s= 2
m

6+6+6 = 33 m
∴ s= 2
cm
s – a = (33 – 25) m = 8 m
= 9 cm s – b = (33 – 23) m = 10 m
s – a = (9 – 6) cm = 3 cm s – c = (33 – 18) m = 15 m
s – b = (9 – 6) cm = 3 cm = s ( s − a )( s − b)( s − c)
s – c = (9 – 6) cm = 3 cm Area of △ABC
= 33 × 8 ×10 ×15 m 2
= s ( s − a )( s − b)( s − c)
∴ Area of △ABC = 60 11 m 2
= 9 × 3 × 3 × 3 cm 2 1
Area of △ABC = × BC × AD
= 15.6 cm 2 (cor. to 3 sig. fig.) 2
1
60 11 m 2 = × 25 m × h
2. a = 30 cm, b = c = 19 cm 2
a +b+c 60 11 × 2
Let s = . h= m
2 25
30 + 19 + 19
∴ s= cm = 15.9 m (cor. to 3 sig. fig.)
2
= 34 cm
s – a = (34 – 30) cm = 4 cm a+b+c
6. Let s = , where a = 10 cm, b = 14 cm and
s – b = (34 – 19) cm = 15 cm 2
s – c = (34 – 19) cm = 15 cm c = 20 cm.
= s ( s − a )( s − b)( s − c) 10 + 14 + 20
∴ s= 2
cm
∴ Area of △ABC
= 34 × 4 ×15 ×15 cm 2 = 22 cm
= 175 cm 2 (cor. to 3 sig. fig.) s – a = (22 – 10) cm = 12 cm
s – b = (22 – 14) cm = 8 cm
s – c = (22 – 20) cm = 2 cm
a+b+c
3. Let s = , where a = 14 cm, b = 15 cm and = s ( s − a )( s − b)( s − c)
2
c = 13 cm. Area of △ABC
= 22 × 12 × 8 × 2 cm 2
14 + 15 + 13
∴ s= 2
cm = 8 66 cm 2
= 21 cm 1
Area of △ABC = × BC × AD
s – a = (21 – 14) cm = 7 cm 2
s – b = (21 – 15) cm = 6 cm 1
8 66 cm = ×10 cm × h
2
s – c = (21 – 13) cm = 8 cm 2
= s ( s − a )( s − b)( s − c) 8 66 × 2
∴ Area of △ABC h= cm
= 21× 7 × 6 × 8 cm 2 10
= 13.0 cm (cor. to 3 sig. fig.)
= 84 cm 2

Level 2
a+b+c 7.
4. Let s = , where a = 9 cm, b = 12 cm and
2
c = 7 cm.
9 + 12 + 7
∴ s= 2
cm

= 14 cm
s – a = (14 – 9) cm = 5 cm
s – b = (14 – 12) cm = 2 cm
s – c = (14 – 7) cm = 7 cm
Join PR.
= s ( s − a )( s − b)( s − c) PR 2 = RS 2 + SP 2 (Pyth. theorem)
∴ Area of △ABC
= 14 × 5 × 2 × 7 cm 2
PR = 6 2 + 8 2 cm
= 31.3 cm 2 (cor. to 3 sig. fig.) = 10 cm
Consider △PQR.

20
Certificate Mathematics in Action Full Solutions 4B

p+q+r BC and DF ⊥ BC.


Let s = , where p = 6 cm, q = 10 cm and
2 ∵ AD // BC
r = 7 cm. ∴ AE = DF
5 + 10 + 7 1
∴ s= cm = × BC × AE
2 Area of △ABC 2
= 11 cm 1
= × BC × DF
s – p = (11 – 5) cm = 6 cm 2
s – q = (11 – 10) cm = 1 cm = area of △BCD
s – r = (11 – 7) cm = 4 cm Consider △ABC.
Area of quadrilateral PQRS
a+b+c
= area of △PQR + area of △PRS Let s = , where a = 12 cm, b = 10 cm and
2
1
= s ( s − p )( s − q )( s − r ) + × RS × PS c = 6 cm.
2 12 + 10 + 6
 1  ∴ s= 2
cm
=  11× 6 × 1× 4 + × 6 × 8  cm 2
 2  = 14 cm
= 40.2 cm 2 (cor. to 3 sig. fig.) s – a = (14 – 12) cm = 2 cm
s – b = (14 – 10) cm = 4 cm
s – c = (14 – 6) cm = 8 cm
8.
Area of △BCD = area of △ABC
= s ( s − a )( s − b)( s − c)
= 14 × 2 × 4 × 8 cm 2
= 29.9 cm 2 (cor. to 3 sig. fig.)

Join OB.
10. Consider △ACD.
Consider △OBA.
∠DAC + ∠ADC + ∠ACD = 180° (∠ sum of △)
OB = OA = 8 cm
AB + OB + OA 37° + ∠ADC + 80° = 180°
Let s1 = . ∠ADC = 63°
2
10 + 8 + 8 By the sine formula,
∴ s1 = cm AC CD
2 =
= 13 cm sin ∠ADC sin ∠DAC
s1 – AB = (13 – 10) cm = 3 cm 8 sin 63°
AC = cm
s1 – OB = (13 – 8) cm = 5 cm sin 37°
s1 – OA = (13 – 8) cm = 5 cm = 11.844 cm
Consider △OCB. BC + AC + AB
OC = OB = 8 cm Let s = .
2
BC + OC + OB 15 + 11.844 + 10
Let s 2 = .
2 ∴ s= 2
cm
5+8+8 = 18.422 cm
∴ s2 = 2
cm
s – BC = (18.422 – 15) cm = 3.422 cm
= 10.5 cm s – AC = (18.422 – 11.844) cm = 6.578 cm
s2 – BC = (10.5 – 5) cm = 5.5 cm s – AB = (18.422 – 10) cm = 8.422 cm
s2 – OC = (10.5 – 8) cm = 2.5 cm Area of △ABC
s2 – OB = (10.5 – 8) cm = 2.5 cm = s ( s − BC )( s − AC )( s − AB)
Area of quadrilateral OCBA
= area of △OBA + area of △OCB = 18.422 × 3.422 × 6.578 × 8.422 cm 2
= s1 ( s1 − AB )( s1 − OB)( s1 − OA) + = 59.1 cm 2 (cor. to 3 sig. fig.)
s 2 ( s 2 − BC )( s 2 − OC )( s 2 − OB )
11.
= ( 13 × 3 × 5 × 5 + 10.5 × 5.5 × 2.5 × 2.5 ) cm 2
= 50.2 cm 2 (cor. to 3 sig. fig.)

9.

Draw
AE ⊥

21
9 Basic Properties of CirclesMore about Trigonometry (II)

Join BE and BD. With the notations in the figure, consider △APH.
Consider △ABE. ∠PAH = 180° − 130°
By the cosine formula, = 50°
BE 2 = AE 2 + AB 2 − 2( AE )( AB ) cos ∠EAB By the cosine formula,
BE = 2 2 + 4 2 − 2( 2)(4) cos 110° km HP 2 = AH 2 + AP 2 − 2( AH )( AP) cos ∠PAH
= 5.047 km HP = 400 2 + 300 2 − 2( 400)(300) cos 50° m
Consider △BCD. = 309 m (cor. to 3 sig. fig.)
By the cosine formula,
∴ Harry is 309 m away from the park.
BD 2 = BC 2 + CD 2 − 2( BC )(CD ) cos ∠BCD
BD = 3.5 2 + 2.8 2 − 2(3.5)(2.8) cos 130° km 3.
= 5.717 km
Consider △BDE.
DE + BE + BD
Let s = .
2
2 + 5.047 + 5.717
∴ s= 2
km

= 6.382 km
s – DE = (6.382 – 2) km = 4.382 km
s – BE = (6.382 – 5.047) km = 1.335 km
s – BD = (6.382 – 5.717) km = 0.665 km
Area of the lake
= area of △ABE + area of △BDE + area of △BCD
1 With the notations in the figure,
= × AB × EA × sin ∠EAB
2 ∠BCD = 180° − 150°
+ s ( s − DE )( s − BE )( s − BD) = 30°
1 Consider △ABC.
+ × BC × CD × sin ∠BCD BC 2 + CA 2 − AB 2
2 cos ∠ACB =
1  2( BC )(CA)
 × 4 × 2 × sin 110° + 
2  48 2 + 60 2 − 55 2
=
=  6.382 × 4.382 × 1.335 × 0.665 +  km 2 2( 48)(60)
 
1  ∠ACB = 60.011°
2 × 3. 5 × 2.8 × sin 130° 
  CA 2 + AB 2 − BC 2
cos ∠CAB =
= 12.5 km 2 (cor. to 3 sig. fig.) 2(CA)( AB)
60 2 + 55 2 − 48 2
=
2(60)(55)
Exercise 9E (p. 196)
∠CAB = 49.103°
Level 1
∠ACD = ∠ACB − ∠BCD
1. By the cosine formula, = 60.011° − 30°
AP 2 + BP 2 − AB 2 = 30.011°
cos ∠APB =
2( AP )( BP) ∠FAC = ∠ACD (alt. ∠s, FA // CD)
12 2 + 15 2 − 7.32 2 = 30.011°
= ∠FAB = ∠FAC + ∠CAB
2(12)(15)
= 30.011° + 49.103°
∠APB = 28.8° (cor. to 3 sig. fig.)
= 79.1° (cor. to 3 sig. fig.)
2. ∴ The true bearing of B from A is 079.1°.

∠DAB = 30° + 90°


4. = 120°
∠ABD = 90° − 48°
= 42°
In △ABD,
∠ADB = 180° − ∠DAB − ∠ABD (∠ sum of △)
= 180° − 120° − 42°
= 18°

22
Certificate Mathematics in Action Full Solutions 4B

By the sine formula, In △ACD,


DB AB AC
= sin ∠ADC =
sin ∠DAB sin ∠ADB AD
DB 50 m AC = AD sin ∠ADC
=
sin 120° sin 18°  200 sin 35° 
50 sin 120° = × sin 50°  m
DB = m  sin 95 ° 
sin 18° = 88.2 m (cor. to 3 sig. fig.)
In △BDC,
∴ The height of the building is 88.2 m.
DC
sin ∠DBC =
DB 7. In △ABD,
DC = DB sin ∠DBC
AB
 50 sin 120°  sin ∠ADB =
= × sin 48°  m AD
 sin 18 °  AB
AD =
= 104 m (cor. to 3 sig. fig.) sin ∠ADB
∴ The height of the hill is 104 m. 40
= m
sin 25°
5. In △ABT, In △ADC,
∠TAB + ∠ATB = ∠TBH (ext. ∠ of △) ∠ADC = 90° − 25° = 65°
25° + ∠ATB = 60° ∠ACD = 90° − 35° = 55°
∠ATB = 35° ∠CAD + ∠ADC + ∠ACD = 180° (∠ sum of △)
By the sine formula, ∠CAD + 65° + 55° = 180°
TB AB ∠CAD = 60°
=
sin ∠TAB sin ∠ATB By the sine formula,
TB 200 m CD AD
= =
sin 25° sin 35° sin ∠CAD sin ∠ACD
200 sin 25° 40
TB = m m
sin 35° CD sin 25°
=
In △TBH, sin 60° sin 55°
TH  40 
sin ∠TBH =  
TB sin 25 °
CD =  × sin 60°  m
TH = TB sin ∠TBH  sin 55° 
 
 200 sin 25°   
= × sin 60°  m
 sin 35°  = 100 m (cor. to 3 sig. fig.)
= 128 m (cor. to 3 sig. fig.)
8.
∴ The height of the tower is 128 m.

6.

Join AC.
In △ABD, With the notations in the figure,
∠BAD + ∠ABD + ∠ADB = 180° (∠ sum of △)
∠PAB = ∠CPA (alt. ∠s, CP // AB)
∠BAD + 35° + 50° = 180°
= 50°
∠BAD = 95° In △APB,
By the sine formula, ∠APB = 110° − 50° = 60°
AD BD ∠PAB + ∠APB + ∠ABP = 180°
= (∠ sum of △)
sin ∠ABD sin ∠BAD 50° + 60° + ∠ABP = 180°
AD 200 m
= ∠ABP = 70°
sin 35° sin 95°
200 sin 35°
AD = m
sin 95°

23
9 Basic Properties of CirclesMore about Trigonometry (II)

By the sine formula, (b)


PA AB
=
sin ∠ABP sin ∠APB
PA 8 km
=
sin 70° sin 60°
8 sin 70°
PA = km
sin 60°
= 8.68 km (cor. to 3 sig. fig.)
∴ The distance between P and A is 8.68 km.
With the notations in the figure,
9. ∠GDP = ∠DPF (alt. ∠s, GD // PF)
= 66°
Consider △DCP.
By the sine formula,
CP DC
=
sin ∠PDC sin ∠DPC
60 km 107.25 km
=
sin ∠PDC sin 99°
60 sin 99°
sin ∠PDC =
107.25
∠PDC = 33.544° or 146.456° (rejected)
With the notations in the figure,
∠GDP + ∠PDC + ∠CDH = 180° (adj. ∠s on st. line)
∠BAL = ∠ALC (alt. ∠s, DA // LC)
66° + 33.544° + ∠CDH = 180°
= 35°
∠BAL + ∠BLA = ∠DBL ∠CDH = 80.5° (cor. to 3 sig. fig.)
(ext. ∠ of △)
35° + ∠BLA = 60° ∴ The compass bearing of C from D is S80.5
°E.
∠BLA = 25°
∠ABL + ∠BAL + ∠BLA = 180° (∠ sum of △) 11. (a)
∠ABL + 35° + 25° = 180°
∠ABL = 120°
By the sine formula,
BA AL
=
sin ∠BLA sin ∠ABL
BA 50 km
=
sin 25° sin 120°
50 sin 25°
BA = km
sin 120°
= 24.400 km
∴ The average speed of the ship With the notations in the figure,
24.400 km ∠ABF = ∠DAB (alt. ∠s, BF // DA)
= = 48°
2h
= 12.2 km/h (cor. to 3 sig. fig.) ∠CBG + ∠CBA + ∠ABF = 180° (adj. ∠s on st. line)
65° + ∠CBA + 48° = 180°
Level 2 ∠CBA = 67°
Consider △ABC.
10. (a) ∠DPC = 66° + 33° By the cosine formula,
= 99° AC 2 = AB 2 + BC 2 − 2( AB)( BC ) cos ∠CBA
DP = (40 × 2) km = 80 km
AC = 12 2 + 20 2 − 2(12)(20) cos 67° km
CP = (30 × 2) km = 60 km
By the cosine formula, = 18.9 km (cor. to 3 sig. fig.)
DC 2 = DP 2 + CP 2 − 2( DP)(CP ) cos ∠DPC ∴ The distance between A and C is 18.9 km.

DC = 80 2 + 60 2 − 2(80)(60) cos 99° km (b) By the sine formula,


= 107 km (cor. to 3 sig. fig.) BC AC
=
∴ The distance between C and D is 107 km. sin ∠CAB sin ∠CBA
20 km 18.880 km
=
sin ∠CAB sin 67°
20 sin 67°
sin ∠CAB =
18.880

24
Certificate Mathematics in Action Full Solutions 4B

∠CAB = 77.192° or 102.808° (rejected) 1


∵ Area of △ABC = × AB × CG
∠CAD = ∠CAB − ∠DAB 2
= 77.192° − 48° 1
× 65 km × CG = 70 × 40 × 25 × 5 km 2
= 29.2° (cor. to 3 sig. fig.) 2

∴ The compass bearing of C from A is N29.2°W. 70 × 40 × 25 × 5 × 2
CG = km
65
12. (a) = 18.2 km (cor. to 3 sig. fig.)
∴ The shortest distance of the ship to island C is
18.2 km.

13. (a)

With the notations in the figure,


∠BCD = 180° − 130° (adj. ∠s on st. line)
= 50°
By the cosine formula,
BC 2 + CA 2 − AB 2
cos ∠ACB =
2( BC )(CA)
With the notations in the figure,
30 2 + 45 2 − 65 2 ∠QAP = ∠AQC (alt. ∠s, PA // QC)
=
2(30)(45) = 20°
∠ACB = 118.782° ∠BAQ = ∠BAP − ∠QAP
∠ACD = ∠ACB − ∠BCD = 55° − 20°
= 118.782° − 50° = 35°
= 68.782° ∠AQB = 50° + 20°
∠FAC = ∠ACD (alt. ∠s, FA // CD) = 70°
= 68.782° ∠ABQ + ∠AQB + ∠BAQ = 180° (∠ sum of △)
By the cosine formula, ∠ABQ + 70° + 35° = 180°
CA 2 + AB 2 − BC 2 ∠ABQ = 75°
cos ∠CAB =
2(CA)( AB ) Consider △APQ.
45 2 + 65 2 − 30 2 PQ
= sin ∠QAP =
2( 45)(65) AQ
∠CAB = 23.861° 10 m
sin 20° =
∠FAB = ∠FAC + ∠CAB AQ
= 68.782° + 23.861° 10
AQ = m
= 92.6° (cor. to 3 sig. fig.) sin 20°
∴ The true bearing of B from A is 092.6°. By the sine formula,
AB AQ
=
(b) The shortest distance of the ship to island C is sin ∠AQB sin ∠ABQ
CG, such that CG ⊥ AB. 10
a+b+c m
Let s = , where a = 30 km, b = 45 km AB sin 20°
2 =
sin 70° sin 75°
and c = 65 km.  10 
30 + 45 + 65  
∴ s= km AB =  sin 20 ° × sin 70°  m
2  sin 75° 
= 70 km  
 
s – a = (70 – 30) km = 40 km
= 28.4 m (cor. to 3 sig. fig.)
s – b = (70 – 45) km = 25 km
s – c = (70 – 65) km = 5 km ∴ The length of the string AB is 28.4 m.

Area of △ABC = s ( s − a )( s − b)( s − c)


= 70 × 40 × 25 × 5 km 2

25
9 Basic Properties of CirclesMore about Trigonometry (II)

(b) Draw BD ⊥ AP. Difference between the two paths


In △ADB, = (5.5271 − 5.3718) km
BD = 0.155 km (cor. to 3 sig. fig.)
sin ∠BAD =
AB ∴ The first path is shorter, the difference between
BD = AB sin ∠BAD the two paths is 0.155 km.
= 28.444 sin 55° m
= 23.3 m (cor. to 3 sig. fig.) 15. ∠HAB = 40° − 15°
= 25°
∴ The height of the balloon above the road AP is
∠HBK = 55° − 15°
23.3 m.
= 40°
14. ∠HAB + ∠AHB = ∠HBK (ext. ∠ of △)
25° + ∠AHB = 40°
∠AHB = 15°
By the sine formula,
HB AB
=
sin ∠HAB sin ∠AHB
HB 10 m
=
sin 25° sin 15°
10 sin 25°
HB = m
sin 15°
Join BA.
With the notations in the figure, Consider △BKH.
∠ABC = ∠BCF (alt. ∠s, BA // FC) ∠BHK = 90° − 55°
= 35° = 35°
Consider △ABC. ∠BHK + ∠HBK + ∠BKH = 180° (∠ sum of △)
By the sine formula, 35° + 40° + ∠BKH = 180°
BC AC ∠BKH = 105°
=
sin ∠BAC sin ∠ABC By the sine formula,
BC 2.3 km HK HB
= =
sin 50° sin 35° sin ∠HBK sin ∠BKH
2.3 sin 50° 10 sin 25°
BC = km HK
m
sin 35° = sin 15°
= 3.0718 km sin 40° sin 105°
∠ABC + ∠ACB + ∠BAC = 180° (∠ sum of △)  10 sin 25° 
 
35° + ∠ACB + 50° = 180° HK =  sin 15° × sin 40°  m
∠ACB = 95°  sin 105° 
 
By the sine formula,  
AB AC = 10.9 m (cor. to 3 sig. fig.)
=
sin ∠ACB sin ∠ABC ∴ The height of the lamppost is 10.9 m.
AB 2.3 km
= ∠PHQ + ∠HQP = ∠HPA
sin 95° sin 35°
16. (a) ∠PHQ + 42° = 65° (ext. ∠ of △)
2.3 sin 95°
AB = km
sin 35° ∠PHQ = 23°
Consider △ADB. Consider △HPQ.
By the cosine formula, By the sine formula,
BD 2 = AB 2 + AD 2 − 2( AB )( AD ) cos ∠BAD HP PQ
=
2 sin ∠HQP sin ∠PHQ
 2.3 sin 95° 
 + 2.8 −
2
 HP 80 m
 sin 35°  =
BD = km sin 42° sin 23°
 2.3 sin 95°  80 sin 42°
2 (2.8) cos 43° HP = m
 sin 35°  sin 23°
= 2.7271 km
= AC + BC
Length of path I
= (2.3 + 3.0718) km
= 5.3718 km
= AD + BD
Length of path II
= (2.8 + 2.7271) km
= 5.5271 km

26
Certificate Mathematics in Action Full Solutions 4B

In △HAP, With the notations in the figure,


HA HC = HA − CA
sin ∠HPA =
HP = HA − KB
HA = HP sin ∠HPA  80 sin 42° 
 × sin 65° − 
 80 sin 42°  sin 23°
= × sin 65°  m = m
 sin 23°   80 sin 35° 
 × sin 60° 
= 124 m (cor. to 3 sig. fig.)  sin 25° 
∠PKQ + ∠KPQ = ∠KQB = 30.135 m
∠PKQ + 35° = 60° (ext. ∠ of △) CK = AB
= 192.187 m
∠PKQ = 25°
In △HCK,
Consider △KPQ. HC
By the sine formula, tan ∠HKC =
CK
KQ PQ
= 30.135 m
sin ∠KPQ sin ∠PKQ =
192.187 m
KQ 80 m ∠HKC = 8.91° (cor. to 3 sig. fig.)
=
sin 35° sin 25°
∴ The angle of elevation of H from K is 8.91
80 sin 35°
KQ = m °.
sin 25°
In △KBQ, 17.
KB
sin ∠KQB =
KQ
KB = KQ sin ∠KQB
 80 sin 35° 
= × sin 60°  m
 sin 25 ° 
= 94.0 m (cor. to 3 sig. fig.)
(b) In △HAP,
AP Join BP.
cos ∠HPA =
HP ∠ABQ + ∠BAP + ∠APQ + ∠BQP = 360°
AP = HP cos ∠HPA ∠ABQ + 70° + 90° + (90° + 38°) = 360°
 80 sin 42°  ∠ABQ = 72°
= × cos 65°  m
 sin 23°  Consider △APB.
In △KBQ, By the cosine formula,
QB BP 2 = AP 2 + AB 2 − 2( AP)( AB ) cos ∠BAP
cos ∠KQB =
KQ BP = 4 2 + 5 2 − 2(4)(5) cos 70° m
QB = KQ cos ∠KQB = 5.2268 m
 80 sin 35°  By the sine formula,
= × cos 60°  m
 sin 25°  AP BP
=
AB = AP + PQ + QB sin ∠ABP sin ∠BAP
4m 5.2268 m
 80 sin 42°  =
 × cos 65° + 80 +  sin ∠ABP sin 70°
sin 23 °
= m 4 sin 70°
 80 sin 35°  sin ∠ABP =
 × cos 60 °  5.2268
 sin 25° 
∠ABP = 45.983° or 134.017° (rejected)
= 192 m (cor. to 3 sig. fig.)
∠PBQ = ∠ABQ − ∠ABP
∴ The distance between A and B is 192 m.
= 72° − 45.983°
(c) = 26.017°
Consider △BPQ.
PQ BP
=
sin ∠PBQ sin ∠BQP
PQ 5.2268 m
=
sin 26.017° sin 128°
5.2268 sin 26.017°
PQ = m
sin 128°
= 2.91 m (cor. to 3 sig. fig.)

27
9 Basic Properties of CirclesMore about Trigonometry (II)

Exercise 9F (p. 209)


Level 1
BG 2 = BC 2 + CG 2 (Pyth. theorem) The angle between AG and plane ADHE is ∠
GAH.
1. (a) BG = 62 + 62 cm AH 2 = AD 2 + DH 2 (Pyth. theorem)
= 72 cm AH = 62 + 52 cm
= 8.49 cm (cor. to 3 sig. fig.)
= 61 cm
BH 2 = BG 2 + HG 2 (Pyth. theorem) HG
tan ∠GAH =
AH
BH = ( 72 ) 2 + 6 2 cm
8
=
= 108 cm 61
= 10.4 cm (cor. to 3 sig. fig.) ∠GAH = 45.7° (cor. to 3 sig. fig.)
∴ The angle between AG and plane ADHE is
(b) The angle between BG and plane ABCD is ∠ 45.7°.
GBC.
GC 3. (a)
tan ∠GBC =
BC
6
=
6
∠GBC = 45°
∴ The angle between BG and plane ABCD is
45°.
The angle between HB and DB is ∠HBD.
(c) The angle between BH and plane BCGF is ∠ BD 2 = AB 2 + AD 2 (Pyth. theorem)
HBG.
HG BD = 7 2 + 52 cm
tan ∠HBG =
BG = 74 cm
6 HD
= tan ∠HBD =
72 BD
∠HBG = 35.3° (cor. to 3 sig. fig.) 4
=
∴ The angle between BH and plane BCGF is 74
35.3°. ∠HBD = 24.9° (cor. to 3 sig. fig.)
∴ The angle between HB and DB is 24.9°.
2. (a)
(b)

The angle between AG and GC is ∠AGC.


AC 2 = AB 2 + BC 2 (Pyth. theorem) The angle between HB and plane CGHD is ∠
BHC.
AC = 8 + 6 cm
2 2

HC 2 = HG 2 + GC 2 (Pyth. theorem)
= 10 cm
AC HC = 7 2 + 42 cm
tan ∠AGC =
CG = 65 cm
10 BC
= tan ∠BHC =
5 HC
∠AGC = 63.4° (cor. to 3 sig. fig.) 5
∴ The angle between AG and GC is 63.4°. =
65
(b) ∠BHC = 31.8° (cor. to 3 sig. fig.)
∴ The angle between HB and plane CGHD is
31.8°.

28
Certificate Mathematics in Action Full Solutions 4B

4. (a) The angle between the planes ACFD and BCFE AQ 2 = AB 2 + BQ 2 (Pyth. theorem)
is ∠ACB.
Consider △ABC. ∵ AQ = AB + BQ2 2

CA2 + BC 2 − AB 2 = AC 2 + CR 2
cos ∠ACB =
2(CA)( BC ) = AR (Pyth. theorem)
102 + 12 2 − 82 ∴ △AQR is an isosceles triangle.
=
2(10)(12) ∴ AM ⊥ QR
3 ∵ PQ = PR = 5 cm
= ∴ △PQR is an isosceles triangle.
4
∠ACB = 41.4° (cor. to 3 sig. fig.) ∴ PM ⊥ QR
∴ The angle between the planes ACFD and ∴ QR is the line of intersection of the planes
AQR and PQR.
BCFE is 41.4°.
∴ The angle between the planes AQR and
(b) The angle between the planes ACFD and BCFE PQR is ∠AMP.
is ∠ACB. Consider △PQM.
Consider △ABC. PQ 2 = PM 2 + QM 2 (Pyth. theorem)
By the sine formula,
PM = 5 2 − 4 2 cm
AB BC
= = 3 cm
sin ∠ACB sin ∠BAC
AP
11 cm 9 cm tan ∠AMP =
= PM
sin ∠ACB sin 35°
10
11 sin 35° =
sin ∠ACB = 3
9
∠AMP = 73.3° (cor. to 3 sig. fig.)
∠ACB = 44.5° (cor. to 3 sig. fig.) or
∴ The angle between the planes AQR and
135° (cor. to 3 sig. fig.) (rejected)
PQR is 73.3°.
∴ The angle between the planes ACFD and
BCFE is 44.5°. 6. (a) In △CBE,
700 m
5. (a) The angle between the planes ABQP and ACRP sin 30° =
BC
is ∠BAC.
700
Consider △ABC. BC = m
sin 30°
AB 2 + AC 2 − BC 2
cos ∠BAC = Consider △DBC.
2( AB )( AC ) BC
cos 55° =
52 + 52 − 8 2 BD
=
2(5)(5) BC
BD =
7 cos 55°
=−
25 700
∠BAC = 106° (cor. to 3 sig. fig.) = sin 30° m
∴ The angle between the planes ABQP and cos 55°
ACRP is 106°. = 2440 m (cor. to 3 sig. fig.)

(b) (b)

The angle between the path BD and plane ABEF


is ∠DBF.
Let M be the mid-point of QR. Consider △DBF.
1 DF
QM = QR sin ∠DBF =
2 BD
1  700 m
=  × 8  cm =
2  2440.826 m
= 4 cm ∠DBF = 16.7° (cor. to 3 sig. fig.)

29
9 Basic Properties of CirclesMore about Trigonometry (II)

∴ The angle between the path BD and plane (b)


ABEF is 16.7°.

7. (a) In △FBE,
60 cm
tan 50° =
BE
60
BE = cm
tan 50°
In △EBC,
EC Let M be the mid-point of AB.
sin 30° = ∵ △VAB is an isosceles triangle.
BE
∴ VM ⊥ AB
EC = BE sin 30°
∵ △NAB is an isosceles triangle.
 60  ∴ NM ⊥ AB
= × sin 30°  cm
 tan 50 °  ∴ AB is the line of intersection of the planes
= 25.2 cm (cor. to 3 sig. fig.) VAB and ABCD.
∴ The angle between the planes VAB and
(b) ABCD is ∠VMN.
6
MN = cm
2
= 3 cm
Consider △VMN.
12
tan ∠VMN =
3
The angle between BF and plane ABCD is ∠ ∠VMN = 76.0° (cor. to 3 sig. fig.)
FBD. ∴ The angle between the planes VAB and
In △FBE, ABCD is 76.0°.
60 cm
sin 50° =
BF 9. (a) The angle between VC and plane ABCD is ∠
60 VCM.
BF = cm 1
sin 50° MB = AB
In △FBD, 2
FD 1 
sin ∠FBD = =  × 8  cm
BF 2 
EC = 4 cm
=
BF In △CMB,
 60  MC 2 = MB 2 + BC 2 ( Pyth. theorem)
 × sin 30°  cm
tan 50 °
=  MC = 4 2 + 6 2 cm
60
cm = 52 cm
sin 50°
∠FBD = 18.7° (cor. to 3 sig. fig.) In △VCM,
VM
∴ The angle between BF and plane ABCD is tan ∠VCM =
18.7°. MC
10 cm
=
8. (a) The angle between VA and plane ABCD is ∠ 52 cm
VAN. ∠VCM = 54.2° (cor. to 3 sig. fig.)
AC = 8 2 + 6 2 cm (Pyth. theorem) ∴ The angle between VC and plane ABCD is
= 10 cm 54.2°.
1
∴ AN = AC = 5 cm (property of rectangle) (b)
2
Consider △VAN.
12
tan ∠VAN =
5
∠VAN = 67.4° (cor. to 3 sig. fig.)
∴ The angle between VA and plane ABCD is
67.4°.

30
Certificate Mathematics in Action Full Solutions 4B

Let N be the mid-point of DC. BD 2 = BC 2 + CD 2 (Pyth. theorem)


The angle between the planes VDC and ABCD 11. (a)
is ∠VNM. BD = 202 + 152 m
In △VNM, = 25 m
VM 1
tan ∠VNM = BK = BD
MN 2

VM 25
= = m
BC 2
10 cm = 12.5 m
=
6 cm Consider △VBK.
∠VNM = 59.0° (cor. to 3 sig. fig.) VK
tan ∠VBK =
∴ The angle between the planes VDC and BK
ABCD is 59.0°. VK = BK tan ∠VBK
= 12.5 tan 15° m
10. (a) The angle between the covers ABCD and BCEF Height of the house = VK + DE
is ∠DCE. = (12.5 tan 15° + 7) m
Consider △DCE. = 10.3 m (cor. to 3 sig. fig.)
CD 2 + CE 2 − DE 2
cos ∠DCE =
2(CD )(CE ) (b)
252 + 252 − 82
=
2( 25)(25)
593
=
625
∠DCE = 18.4° (cor. to 3 sig. fig.)
∴ The angle between the covers ABCD and
BCEF is 18.4°.

(b)
Let M and N be the mid-points of AD and HE
respectively.
The angle between the planes VAD and ADEH is
∠VMN.
∵ △VAD is an isosceles triangle.
∴ VM ⊥ AD
∵ △KAD is an isosceles triangle.
The angle between BD and plane BCEF is ∠ ∴ KM ⊥ AD
DBE. ∴ AD is the line of intersection of the planes
BD 2 = AB 2 + AD 2 (Pyth. theorem) VAD and ABCD.
∴ The angle between planes VAD and
BD = 252 + 20 2 cm
ABCD is ∠VMK.
= 1025 cm 1
KM = FE
BE = BC 2 + CE 2
2
(Pyth. theorem) 2
1 
BE = 20 + 25 cm
2 2
=  × 15  cm
2 
= 1025 cm = 7.5 cm
Consider △BED. Consider △VMK.
By the cosine formula, VK
BD 2 + BE 2 − DE 2 tan ∠VMK =
cos ∠DBE = KM
2( BD )( BE ) 12.5 tan 15°
=
( 1025 ) 2 + ( 1025 ) 2 − 8 2 7.5
=
2( 1025 ) ( 1025 ) ∠VMK = 24.065°
∠VMN = ∠VMK + ∠KMN
993
= = 24.065° + 90°
1025
∠DBE = 14.4° (cor. to 3 sig. fig.) = 114° (cor. to 3 sig. fig.)
∴ The angle between BD and plane BCEF is ∴ The angle between the planes VAD and
14.4°. ADEH is 114°.

31
9 Basic Properties of CirclesMore about Trigonometry (II)

12. (a) Level 2


13. (a) Consider △BDC.
BD 2 = BC 2 + CD 2 (Pyth. theorem)
BD = 120 + 190 cm
2 2

= 50 500 cm
= 225 cm (cor. to 3 sig. fig.)

(b)
Consider △EFG.
EG 2 = EF 2 + FG 2 (Pyth. theorem)
EG = 10 + 18 cm
2 2

= 424 cm
Consider △AEG.
AG 2 = AE 2 + EG 2 (Pyth. theorem)
AG = 14 + 424 cm
2

= 620 cm Join CE and DF.


Consider ABGH. Distance between D and F
∵ BA = GH, AH = BG and ∠ABG = 90° = distance between C and E
∴ ABGH is a rectangle. Consider △BEC.
∴ AK = KG = BK = KH (property of rectangle) By the cosine formula,
KG = KH CE 2 = BC 2 + BE 2 − 2( BC )( BE ) cos ∠CBE
1 CE = 120 2 + 120 2 − 2(120)(120) cos 50° cm
∴ = AG
2
= 101 cm (cor. to 3 sig. fig.)
620
= cm ∴ The distance between D and F is 101 cm.
2
Consider △KGH. (c)
KG 2 + KH 2 − GH 2
cos ∠GKH =
2( KG )( KH )
2 2
 620   620 
  +  − 10 2
 2   2 
   
=
 620  620 
2  
 2  2 
  
21
=
31 Join BD and BF.
∠GKH = 47.4° (cor. to 3 sig. fig.) The angle between BF and DF is ∠BFD.
BF = BD = 50 500 cm
(b) ∵ The angle between the planes KGH and Consider △BFD.
EFGH = the angle between the planes By the cosine formula,
ABGH and EFGH BF 2 + DF 2 − BD 2
∴ The angle between the planes KGH and cos ∠BFD =
2( BF )( DF )
EFGH is ∠BGF.
Consider △BFG. ( 50 500 ) 2 + 101.428 2 − ( 50 500 ) 2
=
BF 2( 50 500 ) (101.428)
tan ∠BGF =
FG = 77.0° (cor. to 3 sig. fig.)
14 The angle between BF and DF is 77.0°.
= ∴
18
∠BGF = 37.9° (cor. to 3 sig. fig.) 14. (a) The angle between the planes AFGD and ABCD
∴ The angle between the planes KGH and is ∠FAB.
EFGH is 37.9°.

32
Certificate Mathematics in Action Full Solutions 4B

Consider △FAB. Consider △DBM.


FB BD 2 = BM 2 + MD 2 (Pyth. theorem)
tan ∠FAB =
AB
MD = 10 − 5 cm
2 2
6
=
9 = 75 cm
∠FAB = 33.7° (cor. to 3 sig. fig.) = 8.66 cm (cor. to 3 sig. fig.)
∴ The angle between the planes AFGD and
ABCD is 33.7°. (b) The angle between AD and plane BCD is ∠
ADM.
(b) Consider △AMD.
By the cosine formula,
AD 2 + MD 2 − AM 2
cos ∠ADM =
2( AD )( MD)
102 + ( 75 ) 2 − ( 75 ) 2
=
2(10)( 75 )
5
=
75
The angle between the planes ABCD and EPQH ∠ADM = 54.7° (cor. to 3 sig. fig.)
is ∠EPA.
∴ The angle between AD and plane BCD is
∵ P is the mid-point of AB.
54.7°.
9
∴ AP = cm
2 (c) BC is the line of intersection of the planes ABC
= 4.5 cm and BCD.
Consider △EAP. ∵ ∠AMB = 90° and ∠DMB = 90°
EA ∴ The angle between the planes ABC and
tan ∠EPA =
AP BCD is ∠AMD.
6 Consider △AMD.
=
4.5 AM 2 + MD 2 − AD 2
∠EPA = 53.1° (cor. to 3 sig. fig.) cos ∠AMD =
2( AM )( MD)
∴ The angle between the planes ABCD and
( 75 ) 2 + ( 75 ) 2 − 10 2
EPQH is 53.1°. =
2( 75 )( 75 )
15. (a) ∵ ABCD is a regular tetrahedron. 1
=
∴ AB = AC = AD = BC = BD = CD = 10 cm 3
∵ M is the mid-point of BC. ∠AMD = 70.5° (cor. to 3 sig. fig.)
BM = MC ∴ The angle between the planes ABC and
1 BCD is 70.5°.
= BC
∴ 2
10 16. (a) Consider △HCA.
= cm
2 HA
tan ∠HCA =
= 5 cm CA
Consider △ABM and △ACM. 20 cm
tan 30° =
AB = AC (given) CA
BM = CM (given) 20
CA = cm
AM = AM (common side) tan 30°
∴ △ABM ≅ △ACM (SSS) = 20 3 cm
∠AMB = ∠AMC Consider △HBA.
(corr. ∠s, ≅ △s) HA
180°
= tan ∠HBA =
2 BA
= 90° 20 cm
tan 60° =
By similar argument, ∠BMD = ∠CMD = 90° BA
Consider △ABM. 20
BA = cm
AB 2 = AM 2 + BM 2 (Pyth. theorem) tan 60°
20
AM = 10 2 − 5 2 cm = cm
3
= 75 cm
= 8.66 cm (cor. to 3 sig. fig.)

33
9 Basic Properties of CirclesMore about Trigonometry (II)

Consider △ACB.
By the cosine formula,
BA2 + BC 2 − CA2
cos ∠ABC =
2( BA)( BC )
2
 20 
 
 3  + 30 − (20 3 )
2 2

=  
 20 
2 (30)

 3
5
=−
12 3
∠ABC = 104° (cor. to 3 sig. fig.)

(b) ∵ M is the mid-point of BC.


BM = MC
∴ 30
= cm
2
= 15 cm
Consider △AMB.
By the cosine formula,
AM 2 = BA2 + BM 2 − 2( BA)( BM ) cos ∠ABC
2
 20 
 
 3  + 15 −
2

AM =   cm
 20 
 
2 (15) cos 103.920°
 3
= 21.016 cm
The angle between HM and plane ABC is ∠
HMA.
Consider △HMA.
HA
tan ∠HMA =
AM
20
=
21.016
∠HMA = 43.6° (cor. to 3 sig. fig.)
∴ The angle between HM and plane ABC is
43.6°.

17. (a) The angle between DB and plane ABC is ∠


DBC.
Consider △DBC.
8
sin ∠DBC =
10
∠DBC = 53.1° (cor. to 3 sig. fig.)
∴ The angle between DB and plane ABC is
53.1°.

(b)

Draw CE ⊥ BA and DE ⊥ BA.


The angle between the planes BDA and ABC is
∠DEC.

34
Certificate Mathematics in Action Full Solutions 4B

Consider △BCD.
BD 2 = BC 2 + CD 2 (Pyth. theorem)
BC = 10 − 8 cm
2 2

= 6 cm
By similar argument, AC = 6 cm
Consider △CBE and △CAE.
BC = AC = 6 cm (proved)
∠BEC = ∠AEC = 90° (as constructed)
CE = CE (common side)
∴ △CBE ≅ △CAE (RHS)
BE = AE (corr. sides, ≅ △s)
Consider △ACB.
AB 2 = AC 2 + CB 2 (Pyth. theorem)
AB = 62 + 6 2 cm
= 72 cm
1
BE = AB
2
72
= cm
2
Consider △BEC.
BC 2 = BE 2 + CE 2 (Pyth. theorem)
2
 72 
CE = 6 2 −   cm
 2 
 
= 18 cm
Consider △DEC.
DC
tan ∠DEC =
CE
8
=
18
∠DEC = 62.1° (cor. to 3 sig. fig.)
∴ The angle between the planes BDA and
ABC is 62.1°.

18. (a)

Join AE.
∠BAE = 90°
Consider △ABE.
BE 2 = AB 2 + AE 2 (Pyth. theorem)
AE = 10 − 6 cm
2 2

= 8 cm
The angle between the planes ABDC and CDFE
is ∠ACE.

35
9 Basic Properties of CirclesMore about Trigonometry (II)

Consider △AEC. Consider △ABN.


AC 2 + CE 2 − AE 2 By the cosine formula,
cos ∠ACE =
2( AC )(CE ) AN 2 + BN 2 − AB 2
cos ∠ANB =
4 2 + 82 − 8 2 2( AN )( BN )
= 2 2
2( 4)(8)  32   32 
  +  − 42
1  2   2 
=
4 =   
 32  32 
∠ACE = 75.5° (cor. to 3 sig. fig.) 2  
 2  2 
∴ The angle between the planes ABDC and   
CDFE is 75.5°. =0
∠ANB = 90°
(b) ∴ The angle between the planes VAN and
VBN is 90°.

(b)

Let G be the projection of E on plane ABDC.


∴ ∠CGE = 90° and ∠BGE = 90°
Consider △CGE.
EG
sin ∠GCE = Let E be the mid-point of AB.
CE
EG ∵ △VAB is an isosceles triangle.
sin 75.522° = ∴ VE ⊥ AB
8 cm
∵ △NAB is an isosceles triangle.
EG = 8 sin 75.522° cm
∴ NE ⊥ AB
The angle between BE and plane ABDC is ∠
EBG. ∴ AB is the line of intersection of the planes
Consider △BEG. VAB and ABCD.
∴ The angle between the planes VAB and
EG
sin ∠EBG = ABCD is ∠VEN.
BE
1
8 sin 75.522° EN = BC
= 2
10
∠EBG = 50.8° (cor. to 3 sig. fig.) 1 
=  × 4  cm
∴ The angle between BE and plane ABDC is 2 
50.8°. = 2 cm
Consider △VEN.
19. (a) Consider △ABC. VN
tan ∠VEN =
AC 2 = AB 2 + BC 2 (Pyth. theorem) EN
6
AC = 4 + 4 cm
2 2 =
2
= 32 cm ∠VEN = 71.6° (cor. to 3 sig. fig.)
1 ∴ The angle between the planes VAB and
AN = AC (property of square)
2 ABCD is 71.6°.
32
= cm (c)
2
BN = AN (property of square)
32
= cm
2
VN is the line of intersection of the planes VAN
and VBN.
∵ AN ⊥ VN and BN ⊥ VN
∴ The angle between the planes VAN and
VBN is ∠ANB.
Let F be the mid-point of DC.

36
Certificate Mathematics in Action Full Solutions 4B

By similar argument as in (b), Consider △EMQ.


∠VFN = 71.6° (cor. to 3 sig. fig.) QE 2 = EM 2 + MQ 2 (Pyth. theorem)
The angle between the planes VAB and VCD is
2 2
∠EVF.  3  3
EM =   −   cm
Consider △VEF.
(∠ sum of △)  2  2
∠EVF + ∠VEN + ∠VFN = 180°
3
∠EVF + 71.565° + 71.565° = 180° = cm
2
∠EVF = 36.9°(cor. to 3 sig. fig.) Consider △BCD.
∴ The angle between the planes VAB and
BD 2 = BC 2 + CD 2 (Pyth. theorem)
VCD is 36.9°.
BD = 6 + 6 cm
2 2

20. (a) Consider △FAP and △HCQ.


= 6 2 cm
∠AFP = ∠CHQ = 90° (given)
1
AP = CQ (given) ND = BD
2
AF = CH (given)
= 3 2 cm
∴ △FAP ≅ △HCQ (RHS)
∴ FP = HQ NR = ND − RD
PE = QE = ND − EM
Consider △PQE.  3
=  3 2 −  cm
PQ 2 = PE 2 + QE 2 (Pyth. theorem)  2
32 cm 2 = 2QE 2 3(2 2 − 1)
= cm
3 2
QE = cm
2 MR = ED = 6 cm
HQ = HE − QE Consider △MNR.
MR
 3  tan ∠MNR =
=  6 −  cm NR
 2 
6 cm
Consider △HCQ. =
3(2 2 − 1)
QC 2 = HQ 2 + HC 2 (Pyth. theorem) cm
2
2
 3  ∠MNR = 65.4° (cor. to 3 sig. fig.)
QC =  6 −  + 6 2 cm
 2  ∴ The angle between the planes APQC and
= 7.14 cm (cor. to 3 sig. fig.) ABCD is 65.4°.

(b) Exercise 9G (p. 216)


Level 1
1. (a) Consider △ACB.
AB
cos ∠ABC =
BC
AB
cos 30° =
100 m
AB = 100 cos 30° m
Consider △TBA.
TA
tan ∠TBA =
Let M and N be the mid-points of PQ and AC AB
respectively. Join MN, ND and ME. Draw a TA
perpendicular PR ⊥ ND. tan 20° =
100 cos 30° m
The angle between the planes APQC and ABCD
TA = 100 cos 30° tan 20° m
is ∠MND.
∵ M is the mid-point of PQ. = 31.5 m (cor. to 3 sig. fig.)
∴ EM ⊥ PQ ∴ The height of the tower TA is 31.5 m.
3
MQ = cm
2

37
9 Basic Properties of CirclesMore about Trigonometry (II)

(b) Consider △ACB. Consider △TAB.


AC TA
sin ∠ABC = tan ∠TBA =
BC AB
AC 500 m
sin 30° = tan 60° =
100 m AB
AC = 100 sin 30° m 500
AB = m
= 50 m tan 60°
Consider △TCA. = 289 m (cor. to 3 sig. fig.)
TA Consider △ACB.
tan ∠TCA =
AC BC 2 = AB 2 + AC 2 (Pyth. theorem)
100 cos 30° tan 20°
= 2
 500   500 
2
50 BC =   +  m
∠TCA = 32.2° (cor. to 3 sig. fig.)  tan 60°   tan 40° 
∴ The angle of elevation of T from C is 32.2 = 662 m (cor. to 3 sig. fig.)
°.
(b)
2. (a) Consider △TAO.
TO
tan ∠TAO =
AO
TO
tan 40° =
50 m
TO = 50 tan 40° m
= 42.0 m (cor. to 3 sig. fig.)
∴ The height of the tower TO is 42.0 m.

(b) Consider △OAB. With the notations in the figure, consider


OB △ACB.
tan ∠BAO =
OA AC 2 + BC 2 − AB 2
OB cos ∠ACB =
tan 50° = 2( AC )( BC )
50 m
OB = 50 tan 50° m 595.88 2 + 662.12 2 − 288.68 2
=
2(595.88)(662.12)
= 59.6 m (cor. to 3 sig. fig.)
∠ACB = 25.849°
∴ The distance between B and O is 59.6 m.
∠BCP = 90° − ∠ACB
(c) Consider △TOB. = 90° − 25.849°
TO = 64.2° (cor. to 3 sig. fig.)
tan ∠TBO =
OB ∴ The compass bearing of B from C is N64.2
50 tan 40° °W.
=
50 tan 50°
∠TBO = 35.1° (cor. to 3 sig. fig.) ∠ACB + ∠CAB + ∠CBA = 180°
4. (a) (∠ sum of △)
∴ The angle of elevation of T from B is 35.1 ∠ACB + 36° + 70° = 180°
°. ∠ACB = 74°
By the sine formula,
3. (a) ∠TCA = 40° AC AB
=
∠TBA = 60° sin ∠CBA sin ∠ACB
Consider △TAC. AC 200 m
=
TA sin 70° sin 74°
tan ∠TCA =
AC 200 sin 70°
AC = m
500 m sin 74°
tan 40° =
AC Consider △XAC.
500 XC
AC = m tan ∠XAC =
tan 40° AC
= 596 m (cor. to 3 sig. fig.) 50
=
200 sin 70°
sin 74°
∠XAC = 14.3° (cor. to 3 sig. fig.)

38
Certificate Mathematics in Action Full Solutions 4B

∴ The angle of elevation of X from A is 14.3 6. (a) Consider △OAB.


°. BA
tan ∠AOB =
OA
(b) Consider △ABC. 1000 m
By the sine formula, tan 30° =
OA
BC AB
= 1000
sin ∠CAB sin ∠ACB OA = m
tan 30°
BC 200 m
= = 1000 3 m
sin 36° sin 74°
Consider △ODC.
200 sin 36°
BC = m CD
sin 74° tan ∠DOC =
Consider △XBC. OD
1000 m
XC tan 20° =
tan ∠XBC = OD
BC
1000
50 OD = m
= tan 20°
200 sin 36°
Consider △AOD.
sin 74°
∠XBC = 22.2° (cor. to 3 sig. fig.) OA
cos ∠AOD =
∵ The angle of depression of B from X OD
= the angle of elevation of X from B 1000
∴ The angle of depression of B from X is tan 30°
=
22.2°. 1000
tan 20°
5. (a) Consider △ABH. ∠AOD = 50.9° (cor. to 3 sig. fig.)
AH
cos ∠BAH = ∴ The compass bearing of C from O is
AB N50.9°E.
AH
cos 60° =
300 m (b) Distance travelled by the helicopter
AH = 300 cos 60° m = BC
= 150 m = 20v m
Consider △LAH. Consider △AOD.
LH OD 2 = OA 2 + AD 2 (Pyth. theorem)
tan ∠LAH =
AH = OA 2 + BC 2
LH 2 2
tan 25° =  1000   1000 
150 m   =  + ( 20v) 2
 tan 20°   tan 30° 
LH = 150 tan 25° m
1000 2 1000 2
= 69.9 m (cor. to 3 sig. fig.) 400v 2 = −
tan 20° tan 2 30°
2

∴ The height of the lighthouse LH is 69.9 m.


 1 1 
v = 2500 − 
(b) Consider △ABH.  tan 2
20 ° tan 2
30 ° 
HB = 107 (cor. to 3 sig. fig.)
sin ∠BAH =
AB
HB 7. (a)
sin 60° =
300 m
HB = 300 sin 60° m
= 260 m (cor. to 3 sig. fig.)

(c) Consider △LHB.


LH
tan ∠LBH =
HB
150 tan 25°
=
300 sin 60°
∠LBH = 15.1° (cor. to 3 sig. fig.) Join AB.
∴ The angle of elevation of L from B is 15.1
°.

39
9 Basic Properties of CirclesMore about Trigonometry (II)

Consider △TAC. 8. (a) Consider △TAO.


TC TO
tan ∠TAC = tan ∠TAO =
AC OA
50 m 20 m
tan 30° = tan 30° =
AC OA
50 20
AC = m OA = m
tan 30° tan 30°
Consider △TBC. Consider △OAC.
TC AC = AB + BC
tan ∠TBC =
BC = (30 + 15) m
50 m = 45 m
tan 20° =
BC OA
cos ∠OAC =
50 AC
BC = m
tan 20° 20
∠ACB = 120° − 90° tan 30°
=
= 30° 45
Consider △ACB. ∠OAC = 39.7° (cor. to 3 sig. fig.)
By the cosine formula, ∴ The true bearing of C from A is 39.7°.
AB 2 = AC 2 + BC 2 − 2( AC )( BC ) cos ∠ACB
2 2 (b) Consider △OAC.
 50   50 
  +  − AC 2 = OA2 + OC 2 (Pyth. theorem)
tan 30°   tan 20° 
AB =  m 2
 50  50   20 
2   cos 30 ° OC = 452 −   m
 tan 30°  tan 20°   tan 30° 
= 75.9 m (cor. to 3 sig. fig.) = 825 m
∴ The distance between A and B is 75.9 m. Consider △TCO.
TO
(b) tan ∠TCO =
OC
20
=
825
∠TCO = 34.8° (cor. to 3 sig. fig.)
∴ The angle of elevation of T from C is 34.8
°.

(c) Consider △OAB.


By the cosine formula,
OB 2 = OA2 + AB 2 − 2(OA)( AB ) cos ∠OAB
2
With the notations in the figure,  20 
  + 30 2 −
∠CBD = ∠ACB (alt. ∠s, DB // CA)  tan 30° 
OB = m
= 30°  20 
2 (30) cos 39.664°
Consider △ACB.  tan 30° 
By the sine formula,
AC AB = 500 m
= Consider △TBO.
sin ∠CBA sin ∠ACB
50 TO
m tan ∠TBO =
tan 30° = 75.931 m OB
sin ∠CBA sin 30° 20
=
50 sin 30° 500
sin ∠CBA = tan 30° ∠TBO = 41.8° (cor. to 3 sig. fig.)
75.931 ∴ The angle of elevation of T from B is 41.8
∠CBA = 34.769° °.
Reflex ∠ABF = 270° + ∠CBD + ∠CBA
= 270° + 30° + 34.769°
= 335° (cor. to 3 sig. fig.)
∴ The true bearing of A from B is 335°.

40
Certificate Mathematics in Action Full Solutions 4B

Level 2 °.
9. (a)
10. (a)

With the notations in the figure, when the man


is at A, let the length of his shadow be x cm.
∵ △GAF ~ △TOF (AAA)
AF GA
=
OF TO
∴ x 1.5
=
With the notations in the figure, 40 + x 14
∠ABC = 70° − 30° = 40° 14 x = 60 + 1.5 x
∠BAD = ∠FBA x = 4.8
= 30° ∴ The length of the shadow of the man is
∠DAC = 180° − 110° 4.8 m when he is at A.
(alt. ∠s. AD // FB)
= 70°
(b)
∠BAC = ∠BAD + ∠DAC
= 30° + 70°
= 100°
Consider △ABC.
∠ABC + ∠BAC + ∠ACB = 180° (∠ sum of △)
40° + 100° + ∠ACB = 180°
∠ACB = 40°
By the sine formula,
BA BC
= With the notations in the figure,
sin ∠ACB sin ∠BAC ∠HDC = 28°
BA 50 m
= Consider △HCD.
sin 40° sin 100° HC
50 sin 40° tan ∠HDC =
BA = m CD
sin 100° 1.5 m
= 32.6 m (cor. to 3 sig. fig.) tan 28° =
CD
1.5
CD = m
(b) Consider △TBA. tan 28°
TA = 2.82 m (cor. to 3 sig. fig.)
tan ∠TBA =
BA ∴ The length of his shadow CD is 2.82 m.
TA
tan 50° =
50 sin 40° (c)
m
sin 100°
50 sin 40° tan 50°
TA = m
sin 100°
= 38.9 m (cor. to 3 sig. fig.)

(c) Consider △ABC.


∵ ∠ACB = ∠ABC = 40°
∴ CA = BA
TA Let J be a point on AB such that OJ ⊥ BA. When
tan ∠TCA =
CA the man is at J, his shadow is the shortest.
TA With the notations in the figure,
=
BA BA 2 = OA 2 + OB 2
= tan ∠TBA (Pyth. theorem)
BA = 40 2 + 25 2 m
∴ ∠TCA = ∠TBA
= 50° = 2225 m
∵ The angle of depression of C from T = 5 89 m
= the angle of elevation of T from C Consider △OAJ and △BAO.
∴ The angle of depression of C from T is 50

41
9 Basic Properties of CirclesMore about Trigonometry (II)

∠OAJ = ∠BAO (common angle) HA = HJ + JA


∠OJA = ∠BOA = 90° = (120 tan 15° + 25) m
∠AOJ = 180° − ∠OAJ − ∠OJA = 57.2 m (cor. to 3 sig. fig.)
= 180° − ∠BAO − ∠BOA
= ∠ABO
∴ △OAJ ~ △BAO (AAA)
OJ OA
=
BO BA
∴ OJ 40
=
25 m 5 89
200
OJ = m
89
Consider △LJK and △TOK.
∵ △LJK ~ △TOK (AAA)
JK LJ
=
OK TO
y 1.5
=
200
+ y 14
∴ 89
300
14 y = + 1.5 y
89
24
y=
89
= 2.54 (cor. to 3 sig. fig.)
∴ The shortest length of his shadow is
2.54 m during his walk from A to B.

11. (a)

Draw a perpendicular from K to HA at J.


JK = AB = 120 m
JA = KB = 25 m
∠HKJ = 15°
Consider △HKJ.
HJ
tan ∠HKJ =
JK
HJ
tan 15° =
120 m
HJ = 120 tan 15° m

42
Certificate Mathematics in Action Full Solutions 4B

(b) Consider △KPB.


KB
tan ∠KPB =
BP
25 m
tan 20° =
BP
25
BP = m
tan 20°
∠ABP = 90° − 50°
= 40°
Consider △APB.
By the cosine formula,
AP 2 = AB 2 + BP 2 − 2( AB)( BP) cos ∠ABP
2
 25 
1202 +   −
AP =  tan 20°  m
 25 
2(120)  cos 40°
 tan 20° 
= 80.559 m
Consider △HPA.
HA
tan ∠HPA =
AP
120 tan 15° + 25
=
80.559
∠HPA = 35.4° (cor. to 3 sig. fig.)
∴ The angle of elevation of H from P is 35.4
°.

(c)

With the notations in the figure, consider


△APB.
By the cosine formula,
AB 2 + AP 2 − BP 2
cos ∠BAP =
2( AB)( AP)
2
 25 
120 2 + 80.559 2 −  
=  tan 20° 
2(120)(80.559)
∠BAP = 33.234°
∠LAP = 90° − ∠BAP
= 90° − 33.234°
= 56.8° (cor. to 3 sig. fig.)
∴ The compass bearing of P from A is S56.8
°E.

43
9 Basic Properties of CirclesMore about Trigonometry (II)

12. (a) Consider △TAG. Consider △PBO.


TG PO
tan ∠TAG = tan ∠PBO =
AG OB
hm hm
tan 25° = tan 60° =
AG OB
h h
AG = m OB = m
tan 25° tan 60°
Consider △TBG. h  3 h 
= m  or m
TG 3  3 
tan ∠TBG =  
BG
Consider △PCO.
hm
tan 15° = PO
BG tan ∠PCO =
OC
h
BG = m hm
tan 15° tan 45° =
OC
(b) Consider △ABG. h
OC = m
AG tan 45°
tan ∠ABG = =h m
BG
h
(b) (i) Consider △OAB.
= tan 25° By the cosine formula,
h
OA 2 + AB 2 − OB 2
tan 15° cos ∠OAB =
2(OA)( AB)
tan 15°
= 2
tan 25°  h 
( 3 h) + 60 − 
2 2

∠ABG = 29.882°
 3 
=
AG 2( 3 h)(60)
sin ∠ABG =
AB
h2
h 3h 2 + 3600 −
= 3
sin 29.882° = tan 25°
450 120 3 h
h = 105 (cor. to 3 sig. fig.) h + 1350  
 or 3 (h + 1350) 
2 2
=
45 3 h  135h 
 
(c)
AC = AB + BC
(ii)
= (60 + 40) m
= 100 m
Consider △OAC.
By the cosine formula,
OA 2 + AC 2 − OC 2
cos ∠OAC =
2(OA)( AC )
( 3 h) 2 + 100 2 − h 2
=
2( 3 h)(100)
With the notations in the figure,
∠HAB = ∠ABG 3 h + 10 000 − h 2
2
=
= 29.9° (cor. to 3 sig. fig.) 200 3 h
∴ The compass bearing of B from A is S29.9 h + 5000 
2
3 ( h 2 + 5000) 
°E. = or
100 3 h  300h 

13. (a) Consider △PAO.
(c)
PO
tan ∠PAO =
OA
hm
tan 30° =
OA
h
OA = m
tan 30°
= 3 hm

44
Certificate Mathematics in Action Full Solutions 4B

(c)
With the notations in the figure,
∵ cos ∠OAB = cos ∠OAC
h 2 + 1350 h 2 + 5000
=
45 3 h 100 3 h
∴ 20 h 2 + 27 000 = 9 h 2 + 45 000

11 h 2 = 18 000
h = 40.5 (cor. to 3 sig. fig.)
With the notations in the figure,
OA = 3 h m ∠BAF = 90° − 85°
= 70.065 m = 5°
OC = h m ∠DCA = ∠CAF (alt. ∠s, DC // AF )
= 40.452 m = ∠CAB + ∠BAF
Consider △OAC.
= 47° + 5°
By the cosine formula,
= 52°
OA 2 + OC 2 − AC 2
cos ∠AOC = Consider △CAB.
2(OA)(OC )
AC 2 + BC 2 − AB 2
70.065 2 + 40.452 2 − 100 2 cos ∠ACB =
= 2( AC )( BC )
2(70.065)(40.452)
712 + 56.2312 − 70 2
∠AOC = 127.548° =
∠DOC = 180° − ∠AOC 2(71)(56.231)
= 180° − 127.548° ∠ACB = 65.566°
∠GCB = 270° − ∠DCA − ∠ACB
= 52.5° (cor. to 3 sig. fig.)
= 270° − 52° − 65.566°
∴ The compass bearing of C from O is
= 152° (cor. to 3 sig. fig.)
N52.5°E.
∴ The true bearing of B from C is 152°.
14. (a) Consider △XAC.
XC Revision Exercise 9 (p. 221)
tan ∠XAC =
AC Level 1
XC 1 
tan 35° = =  × 30 × 24 × sin 56°  cm 2
71 m 1. (a) Area of △ABC 2 
XC = 71 tan 35° m
= 298 cm 2 (cor. to 3 sig. fig.)
= 49.7 m (cor. to 3 sig. fig.)
∴ The height of the tower XC is 49.7 m.
1 
=  × 3.7 × 6 × sin 102°  cm 2
(b) Area of △ABC  2 
(b) ∠CAB = 85° − 38°
= 47° = 10.9 cm 2 (cor. to 3 sig. fig.)
Consider △CAB.
By the cosine formula, a+b+c
(c) Let s = , where a = 5.1 cm, b = 7.4 cm
BC 2 = AB 2 + AC 2 − 2( AB)( AC ) cos ∠CAB 2
and c = 4.1 cm.
BC = 70 2 + 712 − 2(70)(71) cos 47° m 5.1 + 7.4 + 4.1
= 56.231 m ∴ s= 2
cm
Consider △XBC. = 8.3 cm
XC s – a = (8.3 – 5.1) cm = 3.2 cm
tan ∠XBC =
BC s – b = (8.3 – 7.4) cm = 0.9 cm
71 tan 35° s – c = (8.3 – 4.1) cm = 4.2 cm
=
56.231 = s ( s − a )( s − b)( s − c)
∠XBC = 41.5° (cor. to 3 sig. fig.) ∴ Area of △ABC
= 8.3 × 3.2 × 0.9 × 4.2 cm 2
∴ The angle of elevation of X from B is 41.5
= 10.0 cm 2 (cor. to 3 sig. fig.)
°.

a+b+c
(d) Let s = , where a = 31 cm, b = 29 cm
2
and c = 20 cm.

45
9 Basic Properties of CirclesMore about Trigonometry (II)

31 + 29 + 20 Consider △AGF.
∴ s= 2
cm
AG 2 = AF 2 + FG 2
(Pyth. theorem)
= 40 cm
AG = 4 2 + 6 2 cm
s – a = (40 – 31) cm = 9 cm
s – b = (40 – 29) cm = 11 cm = 52 cm
s – c = (40 – 20) cm = 20 cm = 7.21 cm (cor. to 3 sig. fig.)
= s ( s − a)( s − b)( s − c) Consider △GEH.
∴ Area of △ABC
= 40 × 9 ×11× 20 cm 2
EG 2 = EH 2 + HG 2
(Pyth. theorem)
= 281 cm 2 (cor. to 3 sig. fig.) EG = 6 2 + 32 cm
= 45 cm
a+b+c
2. (a) Let s = , where a = 35 cm, b = 31 cm = 6.71 cm (cor. to 3 sig. fig.)
2
and c = 24 cm.
35 + 31 + 24 AE + AG + EG
(b) Let s = .
∴ s= 2
cm 2
= 45 cm 5 + 52 + 45
s – a = (45 – 35) cm = 10 cm ∴ s= 2
cm
s – b = (45 – 31) cm = 14 cm = 9.460 cm
s – c = (45 – 24) cm = 21 cm s – AE = (9.460 – 5) cm = 4.460 cm
= s ( s − a )( s − b)( s − c) s – AG = (9.460 – 52 ) cm = 2.249 cm
Area of △ABC
= 45 ×10 ×14 × 21 cm 2 s – EG = (9.460 – 45 ) cm = 2.752 cm
= 210 3 cm 2 Area of △AEG
1 = s ( s − AE )( s − AG )( s − EG )
Area of △ABC = × BC × AD
2
= 9.460 × 4.460 × 2.249 × 2.752 cm 2
1
210 3 cm = × 35 cm × h
2
= 16.2 cm 2 (cor. to 3 sig. fig.)
2
210 3 × 2
h= cm ∠ADB + ∠ADC = 180°
35 4. (adj. ∠s on st. line)
∠ADB + 65° = 180°
= 20.8 cm (cor. to 3 sig. fig.)
∠ADB = 115°
Area of △ABC
a+b+c
(b) Let s = , where a = 42 cm, b = 50 cm = area of △ABD + area of △ADC
2
1 1
and c = 80 cm. = × AD × BD × sin ∠ADB + × AD × DC × sin ∠ADC
42 + 50 + 80 2 2
∴ s= 2
cm 1 1 
=  ×12 × 8 × sin 115° + ×12 × 5 × sin 65°  cm 2
= 86 cm 2 2 
s – a = (86 – 42) cm = 44 cm = 70.7 cm (cor. to 3 sig. fig.)
2

s – b = (86 – 50) cm = 36 cm
s – c = (86 – 80) cm = 6 cm
5. (a) BC = AD = 20 cm(opp. sides of // gram)
= s ( s − a )( s − b)( s − c) Consider △ABC.
Area of △ABC By the sine formula,
= 86 × 44 × 36 × 6 cm 2
BC AB
= 24 1419 cm 2 =
sin ∠BAC sin ∠ACB
1 20 cm 16 cm
Area of △ABC = × BC × AD =
2 sin ∠BAC sin 44°
1 20 sin 44°
24 1419 cm = × 42 cm × h
2
sin ∠BAC =
2 16
24 1419 × 2 ∠BAC = 60.264° or 119.736° (rejected)
h= cm
42 ∠ABC + ∠ACB + ∠BAC = 180° (∠ sum of △)
= 43.1 cm (cor. to 3 sig. fig.) ∠ABC + 44° + 60.264° = 180°
∠ABC = 75.736°
3. (a) Consider △AEF.
AE 2 = AF 2 + FE 2
(Pyth. theorem)
AE = 4 2 + 32 cm
= 5 cm

46
Certificate Mathematics in Action Full Solutions 4B

Area of ABCD A + B + C = 180° (∠ sum of △)


= 2 × area of △ABC 120° + 25.659° + C = 180°
1 C = 34.341°
= 2 × × AB × BC × sin ∠ABC
2 By the sine formula,
 1  c a
=  2 × × 16 × 20 × sin 75.736°  cm 2 =
 2  sin C sin A
= 310 cm 2 (cor. to 3 sig. fig.) x cm 10 cm
=
sin 34.341° sin 120°
(b) 10 sin 34.341°
x=
sin 120°
= 6.51 (cor. to 3 sig. fig.)

(c) By the cosine formula,


a 2 = b 2 + c 2 − 2bc cos A
Join BD. x = 5 2 + 4 2 − 2(5)(4) cos 60°
DC = AB = 11 cm
Area of ABCD = 21
= 2 × area of △BCD = 4.58 (cor. to 3 sig. fig.)
1
= 2 × × BC × DC × sin ∠BCD a 2 + b2 − c2
2 cos C =
2ab
 1 
=  2 × ×15 ×11× sin 125°  cm 2 ( 21 ) 2 + 5 2 − 4 2
 2  cos θ =
= 135 cm 2 (cor. to 3 sig. fig.) 2( 21 )(5)
3
=
1 21
6. Area of △ABC = × AB × BC × sin B θ = 49.1° (cor. to 3 sig. fig.)
2
1 (d) By the sine formula,
5 = × x × ( x + 1) × sin 30°
2 c a
=
x + x − 20 = 0
2 sin C sin A
( x − 4)( x + 5) = 0 2.8 cm 3.2 cm
=
x = 4 or x = −5 (rejected) sin θ sin 66°
2.8 sin 66°
sin θ =
3.2
7. (a) By the cosine formula,
θ = 53.1° (cor. to 3 sig. fig.) or
a2 + c2 − b2
cos B = 127° (cor. to 3 sig. fig.) (rejected)
2ac
8.9 2 + 5.2 2 − 5 2 A + B + C = 180° (∠ sum of △)
cos θ =
2(8.9)(5.2) 66° + B + 53.068° = 180°
θ = 28.6° (cor. to 3 sig. fig.) B = 60.932°
By the sine formula,
b2 + c2 − a2 b a
cos A = =
2bc sin B sin A
5 + 5.2 2 − 8.9 2
2
x cm 3.2 cm
cos φ = =
2(5)(5.2) sin 60.932° sin 66°
φ = 122° (cor. to 3 sig. fig.) 3.2 sin 60.932°
x=
sin 66°
(b) By the sine formula, = 3.06 (cor. to 3 sig. fig.)
b a
=
sin B sin A A + B + C = 180°
5 cm
=
10 cm 8. (a) 42° + B + 68° = 180° (∠ sum of △)
sin θ sin 120° B = 70°
5 sin 120°
sin θ =
10
θ = 25.7° (cor. to 3 sig. fig.) or
154° (cor. to 3 sig. fig.) (rejected)

47
9 Basic Properties of CirclesMore about Trigonometry (II)

By the sine formula, (d) By the sine formula,


c a a b
= =
sin C sin A sin A sin B
c 15 cm 22 cm 26 cm
= =
sin 68° sin 42° sin A sin 120°
15 sin 68° 22 sin 120°
c= cm sin A =
sin 42° 26
= 20.8 cm (cor. to 3 sig. fig.) A = 47.1° (cor. to 3 sig. fig.) or
b a 133° (cor. to 3 sig. fig.) (rejected)
=
sin B sin A A + B + C = 180°
b 15 cm 47.121° + 120° + C = 180°
=
sin 70° sin 42° C = 12.9° (cor. to 3 sig. fig.)
15 sin 70°
b= cm By the sine formula,
sin 42° c b
= 21.1 cm (cor. to 3 sig. fig.) =
sin C sin B
c 26 cm
A + B + C = 180° =
∠ △) sin 12.879° sin 120°
(b) A + 38° + 100° = 180° ( sum of 26 sin 12.879°
c= cm
A = 42° sin 120°
By the sine formula, = 6.69 cm (cor. to 3 sig. fig.)
a b
=
sin A sin B (e) By the cosine formula,
a 28 cm b2 + c2 − a2
= cos A =
sin 42° sin 38° 2bc
28 sin 42° 35 2 + 20 2 − 30 2
a= cm =
sin 38° 2(35)(20)
= 30.4 cm (cor. to 3 sig. fig.)
29
=
c b 56
=
sin C sin B A = 58.8° (cor. to 3 sig. fig.)
c 28 cm a2 + c2 − b2
= cos B =
sin 100° sin 38° 2ac
28 sin 100°
c= cm 30 2 + 20 2 − 35 2
sin 38° =
2(30)(20)
= 44.8 cm (cor. to 3 sig. fig.)
1
=
16
(c) By the cosine formula,
B = 86.4° (cor. to 3 sig. fig.)
c 2 = a 2 + b 2 − 2ab cos C
A + B + C = 180° (∠ sum of △)
c = 12 2 + 18 2 − 2(12)(18) cos 54° cm
58.811° + 86.417° + C = 180°
= 14.6 cm (cor. to 3 sig. fig.) C = 34.8° (cor. to 3 sig. fig.)
b2 + c2 − a 2
cos A =
2bc 9. (a) Consider △ACD.
18 + 14.6312 − 12 2
2 By the cosine formula,
= AC 2 + CD 2 − AD 2
2(18)(14.631) cos ∠ACD =
A = 41.6° (cor. to 3 sig. fig.) 2( AC )(CD )
A + B + C = 180°(∠ sum of △) 92 + 42 − 62
=
41.569° + B + 54° = 180° 2(9)(4)
B = 84.4° (cor. to 3 sig. fig.) 61
=
72
∠ACD = 32.1° (cor. to 3 sig. fig.)

∠ACB + ∠ACD = 180°


(b) (adj. ∠s on st. line)
∠ACB + 32.089° = 180°
∠ACB = 147.911°

48
(∠ sum of △)
Certificate Mathematics in Action Full Solutions 4B

Consider △ABC. Consider △AKD.


By the sine formula, ∠AKB = ∠ADK + ∠DAK
AC AB (ext. ∠ of △)
= 110° = ∠ADK + 75°
sin ∠ABD sin ∠ACB ∠ADK = 35°
9 cm 13 cm By the sine formula,
=
sin ∠ABD sin 147.911° KD AK
9 sin 147.911° =
sin ∠ABD = sin ∠DAK sin ∠ADK
13 5 sin 30°
∠ABD = 21.6° (cor. to 3 sig. fig.) or cm
KD
= sin 40°
158° (cor. to 3 sig. fig.) (rejected) sin 75° sin 35°
5 sin 30° sin 75°
KD = cm
10. Consider △ABC. sin 35° sin 40°
By the cosine formula, = 6.55 cm (cor. to 3 sig. fig.)
AC 2 = AB 2 + BC 2 − 2( AB )( BC ) cos ∠ABC
12.
AC = 12 2 + 18 2 − 2(12)(18) cos 60° cm
= 252 cm
= 15.9 cm (cor. to 3 sig. fig.)
∵ ABCD is a parallelogram.
∴ DC = AB = 12 cm and ∠BCD = 180° – 60° =
120°
Consider △BCD.
With the notations in the figure,
By the cosine formula,
∠ABD = ∠CAB (alt. ∠s, DB // AC)
BD 2 = BC 2 + DC 2 − 2( BC )( DC ) cos ∠BCD
= 54°
BD = 18 2 + 12 2 − 2(18)(12) cos 120° cm ∠ABC = ∠CBD − ∠ABD
= 684 cm = 78° − 54°
= 26.2 cm (cor. to 3 sig. fig.) = 24°
Consider △ACB.
∠CAB + ∠ACB + ∠ABC = 180°
11. (a) Consider △BCK. (∠ sum of △)
∠AKB = ∠BCK + ∠KBC (ext. ∠ of △) 54° + ∠ACB + 24° = 180°
= 30° + 80° ∠ACB = 102°
By the sine formula,
= 110°
AB AC
Consider △ABK. =
sin ∠ACB sin ∠ABC
By the sine formula,
AB 400 m
AB KB =
= sin 102° sin 24°
sin ∠AKB sin ∠KAB
400 sin 102°
AB 5 cm AB = m
= sin 24°
sin 110° sin 40°
BC AC
5 sin 110° =
AB = cm sin ∠CAB sin ∠ABC
sin 40°
BC 400 m
= 7.31 cm (cor. to 3 sig. fig.) =
sin 54° sin 24°
400 sin 54°
(b) Consider △ABK. BC = m
sin 24°
∠KAB + ∠ABK + ∠AKB = 180°
(∠ sum of △) Total distance ran by the man
40° + ∠ABK + 110° = 180° = AB + BC
∠ABK = 30°
 400 sin 102° 400 sin 54° 
By the sine formula, = + m
AK KB  sin 24° sin 24° 
= = 1760 m (cor. to 3 sig. fig.)
sin ∠ABK sin ∠KAB
AK 5 cm
= ∠ADB = 60° − 25°
sin 30° sin 40° 13.
5 sin 30° = 35°
AK = cm
sin 40° ∠DAB = 90° − 60°
= 30°

49
9 Basic Properties of CirclesMore about Trigonometry (II)

Consider △ADB. km
By the sine formula, Hence, at 2 p.m., boys X and Y reach points G
AB DB and H respectively, as shown in the figure.
= ∠GOH = 180° − 60°
sin ∠ADB sin ∠DAB
AB 9m = 120°
= OG = 10 km
sin 35° sin 30°
9 sin 35° OH = 2 km
AB = m
sin 30° Consider △OGH.
= 10.3 m (cor. to 3 sig. fig.) By the cosine formula,
∴ The height of the flagstaff is 10.3 m. GH 2 = OG 2 + OH 2 − 2(OG )(OH ) cos ∠GOH
GH = 10 2 + 2 2 − 2(10)(2) cos 120° km
14.
= 124 km
= 11.1 km (cor. to 3 sig. fig.)
∴ The distance between the two boys at 2 p.m.
is 11.1 km.

With the notations in the figure, (b)


∠ACB = ∠FAC (alt. ∠s, BC // AF)
= 42°
Consider △ACB.
AB
sin ∠ACB =
AC
50 m
sin 42° =
AC
50
AC = m
sin 42°
∠DAC = ∠FAC − ∠FAD With the notations in the figure, by the cosine
= 42° − 20° formula,
= 22° OG 2 + GH 2 − OH 2
cos ∠OGH =
∠ADC = ∠FAD (alt. ∠s, AF // CD) 2(OG )(GH )
= 20° 10 2 + ( 124 ) 2 − 2 2
=
Consider △ACD. 2(10)( 124 )
By the sine formula, 11
CD AC =
= 124
sin ∠DAC sin ∠ADC
∠OGH = 8.948°
50
m ∠KGO = 90° − 60°
CD sin 42°
=
sin 22° sin 20° = 30°
50 sin 22° ∠KGH = ∠KGO + ∠OGH
CD = m
sin 20° sin 42° = 30° + 8.948°
= 81.8 m (cor. to 3 sig. fig.) = 38.9° (cor. to 3 sig. fig.)
∴ The distance between the two landmarks is ∴ The true bearing of Y from X at 2 p.m. is
81.8 m. 038.9°.
15. (a) 16. (a)

Distance travelled by boy X = (5 × 2) km = 10


km
Distance travelled by boy Y = (6 × 2) km = 12

50
Certificate Mathematics in Action Full Solutions 4B

With the notations in the figure, Consider △DBE.


∠ABF = ∠DAB (alt. ∠s, BF // DA) DE
sin ∠DBE =
= 60° BD
∠CBF = 180° − ∠GBC (adj. ∠s on st. line) 40 sin 10°
=
= 180° − 135° 40
= 45° cos 51.340°
∠ABC = ∠ABF + ∠CBF ∠DBE = 6.23° (cor. to 3 sig. fig.)
= 60° + 45° ∴ The angle between BD and plane BCEF is
= 105° 6.23°.
Consider △ACB.
18. (a)
By the cosine formula,
AC 2 = AB 2 + BC 2 − 2( AB)( BC ) cos ∠ABC
AC = 4 2 + 5 2 − 2( 4)(5) cos 105° km
= 7.17 km (cor. to 3 sig. fig.)

AB 2 + AC 2 − BC 2
cos ∠BAC =
2( AB)( AC )
(b)
4 2 + 7.166 2 − 5 2 Join AC and BD.
= AQ is the projection of AP on plane ABCD.
2( 4)(7.166)
The angle that PA makes with the base ABCD is
∠BAC = 42.374°
∠PAQ.
∠DAB + ∠BAC + ∠HAC = 180° (adj. ∠s on st. line)
Consider △ABC.
60° + 42.374° + ∠HAC = 180°
AC 2 = AB 2 + BC 2 (Pyth. theorem)
∠HAC = 77.6° (cor. to 3 sig. fig.)
AC = 12 2 + 16 2 cm
∴ The compass bearing of C from A is S77.6
°E. = 20 cm
1
AQ = AC
17. (a) The angle between BD and BA is ∠ABD. 2 (property of rectangle)
Consider △ABD. 1 
=  × 20  cm
AD 2 
tan ∠ABD =
AB = 10 cm
50 PQ = GA = 5 cm
=
40 Consider △PAQ.
∠ABD = 51.3° (cor. to 3 sig. fig.) PQ
tan ∠PAQ =
∴ The angle between BD and BA is 51.3°. AQ
5
(b) The angle between BD and plane BCEF is ∠ =
10
DBE. ∠PAQ = 26.6° (cor. to 3 sig. fig.)
Consider △ABD.
∴ The angle that PA makes with the base
AB
cos ∠ABD = ABCD is 26.6°.
BD
40 cm (b)
cos 51.340° =
BD
40
BD = cm
cos 51.340°
Consider △DCE.
DE
sin ∠DCE =
DC
DE
sin 10° =
40 cm Join PA and PB.
DE = 40 sin 10° cm Let M and N be the mid-points of AB and GF
respectively.
The angle between the planes PAB and ABFG is
∠PMN.

51
9 Basic Properties of CirclesMore about Trigonometry (II)

1 20. (a) Consider △TBP.


PN = EF
2 TP
tan ∠TBP =
1  BP
=  × 16  cm
 2  45 m
tan 40° =
= 8 cm BP
Consider △PMN. 45
BP = m
PN tan 40°
tan ∠PMN = Consider △TAP.
NM
8 TP
= tan ∠TAP =
5 AP
∠PMN = 58.0° (cor. to 3 sig. fig.) 45 m
tan 30° =
∴ The angle between the planes PAB and AP
ABFG is 58.0°. 45
AP = m
tan 30°
19. (a) Consider △ABC. ∠BPA = 200° − 150°
∠ABC + ∠ACB + ∠BAC = 180° (∠ sum of △) = 50°
60° + 72° + ∠BAC = 180° Consider △BAP.
∠BAC = 48° By the cosine formula,
By the sine formula, AB 2 = BP 2 + AP 2 − 2( BP )( AP ) cos ∠BPA
AB BC 2 2
=  45   45 
sin ∠ACB sin ∠BAC   +  −
 tan 40°   tan 30° 
AB 14 cm AB = m
=  45  45 
sin 72° sin 48° 2   cos 50°
14 sin 72°  tan 40°  tan 30° 
AB = cm
sin 48° = 59.8 m (cor. to 3 sig. fig.)
= 17.9 cm (cor. to 3 sig. fig.)
Consider △PBA. (b)
PA
tan ∠PBA =
AB
PA
tan 30° =
14 sin 72°
cm
sin 48°
14 sin 72° tan 30°
PA = cm
sin 48°
= 10.3 cm (cor. to 3 sig. fig.)

(b) Area of △ABC


1
= × AB × BC × sin ∠ABC With the notations in the figure,
2
∠BPQ = 200° – 180° = 20°
1 
=  ×17.917 ×14 × sin 60°  cm 2 ∠RBP = ∠BPQ (alt. ∠s, RB // PQ)
2 
= 20°
= 109 cm 2 (cor. to 3 sig. fig.)
Consider △BAP.
By the cosine formula,
(c) Volume of the tetrahedron PBCA BP 2 + AB 2 − AP 2
1 cos ∠PBA =
= × area of △ABC × PA 2( BP)( AB)
3 2 2
1   45   45 
=  × 108.616 ×10.344  cm 3   + 59.8112 −  
tan 40 °  tan 30° 
 3  = 
 45 
= 375 cm 3 (cor. to 3 sig. fig.) 2 (59.811)
 tan 40° 
∠PBA = 86.618°
∠RBA = ∠RBP + ∠PBA
= 20° + 86.618°
= 107° (cor. to 3 sig. fig.)
∴ The true bearing of A from B is 107°.

52
Certificate Mathematics in Action Full Solutions 4B

21. (a) 22. Since A is an obtuse angle, cos A < 0.


By the cosine formula,
b2 + c2 − a2
cos A =
2bc
20 2 + c 2 − 30 2
=
2(20)(c )
c 2 − 500
=
40c
cos A < 0

With the notations in the figure, c 2 − 500
AG = BH = 800 m <0
40c
Consider △ACG. ∵ c>0
AG
tan ∠ACG = c 2 − 500 < 0
CG ∴
800 m c 2 < 500
tan 50° =
CG 0 < c < 500
800 ∴ A possible integral value of c is 15 cm. (or any
CG = m
tan 50° other reasonable answers)
Consider △CHG.
GH 23.
tan ∠HCG =
CG
GH
tan 70° =
800
m
tan 50°
800 tan 70°
GH = m
tan 50°
∴ The speed of the aeroplane Assume the required triangle is ABC.
∵ Area of △ABC = 24 cm2
800 tan 70°
m Take C = 45°, we have
= tan 50° 1
5s ab sin C = 24 cm 2
= 369 m/s (cor to 3 sig. fig.) 2
1
ab sin 45° = 24 cm 2
(b) Consider △CHG. 2
CG 1  1 
cos ∠HCG = ab  = 24 cm 2
CH 2  2 
800 ab = 48 2 cm 2
m
cos 70° = tan 50° Take a = 8 cm, then
CH
48 2
800 b= cm
CH = m 8
tan 50° cos 70°
= 6 2 cm
Consider △BCH.
By the cosine formula,
BH
tan ∠BCH = c 2 = a 2 + b 2 − 2ab cos C
CH
800 c = 8 2 + (6 2 ) 2 − 2(8)(6 2 ) cos 45° cm
=
800
= 2 10 cm
tan 50° cos 70°
∠BCH = 22.2° (cor. to 3 sig. fig.) a + c = [8 2 + (2 10 ) 2 ] cm 2
2 2

∴ The angle of elevation of B from C is 22.2 = 104 cm 2


°.
b 2 = (6 2 ) 2 cm 2
= 72 cm 2
∴ a + c2 ≠ b2
2

∴ ∠B ≠ 90°
i.e. △ABC is not a right-angled triangle.
∴ The lengths of the sides of a triangle which
satisfies the required two conditions are 8 cm,
6 2 cm and 2 10 cm. (or any other
reasonable answers)

53
9 Basic Properties of CirclesMore about Trigonometry (II)

Level 2 Consider △ABD.


BD = (7 + 5) cm = 12 cm
24. (a) Consider △ACD. By the cosine formula,
∠CAD + ∠ACD + ∠ADC = 180° (∠ sum of △) AD 2 = AB 2 + BD 2 − 2( AB )( BD) cos ∠ABD
45° + ∠ACD + 90° = 180°  11 
AD = 8 2 + 12 2 − 2(8)(12)  cm
∠ACD = 45°  16 
Consider △ABC.
∠BAC + ∠ABC = ∠ACD x = 76
(ext. ∠ of △)
∠BAC + 30° = 45° = 8.72 (cor. to 3 sig. fig.)
∠BAC = 15°
By the sine formula, (d) Consider △AMC.
AC BC AC = AM = 8 cm (given)
= ∠ACM = ∠AMC (base ∠s, isos. △)
sin ∠ABC sin ∠BAC
∠MAC + ∠AMC + ∠ACM = 180° (∠ sum of △)
AC 25 cm
= 40° + 2∠ACM = 180°
sin 30° sin 15°
25 sin 30° ∠ACM = 70°
AC = cm By the cosine formula,
sin 15°
Consider △ACD. MC 2 = AM 2 + AC 2 − 2( AM )( AC ) cos ∠MAC
AD MC = 8 2 + 8 2 − 2(8)(8) cos 40° cm
cos ∠CAD =
AC
= 5.472 cm
x cm
cos 45° = BM = MC = 5.472 cm
25 sin 30°
cm
sin 15° BC = BM + MC
25 sin 30° = (5.472 + 5.472) cm
x = cos 45° ×
sin 15° = 10.944 cm
= 34.2 (cor. to 3 sig. fig.) Consider △ABC.
By the cosine formula,
(b) Consider △ACD. AB 2 = AC 2 + BC 2 − 2( AC )( BC ) cos ∠ACB
∠ADC + ∠DAC + ∠ACD = 180° (∠ sum of △)
AB = 8 2 + 10.944 2 − 2(8)(10.944) cos 70° cm
∠ADC + 40° + 68° = 180°
x = 11.1 (cor. to 3 sig. fig.)
∠ADC = 72°
By the sine formula,
AC AD 25. Consider △AOC.
= ∠OAC + ∠AOC + ∠ACO = 180° (∠ sum of △)
sin ∠ADC sin ∠ACD
AC 7 cm 30° + 50° + ∠ACO = 180°
=
sin 72° sin 68° ∠ACO = 100°
7 sin 72° By the sine formula,
AC = cm
sin 68° OC OA
=
Consider △ABC. sin ∠OAC sin ∠ACO
By the cosine formula, OC 13 cm
BC 2 = AB 2 + AC 2 − 2( AB)( AC ) cos ∠BAC =
sin 30° sin 100°
 7 sin 72° 
2 13 sin 30°
82 +   − OC = cm
sin 100°
BC =  sin 68°  cm Area of the shaded region
 7 sin 72°  = area of sector OAB − area of △AOC
2(8)  cos 80°
 sin 68°   50° 1 13 sin 30° 
=  π ×13 2 × − ×13 × × sin 50°  cm 2
x = 9.78 (cor. to 3 sig. fig.)  360° 2 sin 100° 
= 40.9 cm 2 (cor. to 3 sig. fig.)
(c) Consider △ABC.
By the cosine formula,
26. Let OC = x cm.
AB 2 + BC 2 − AC 2
cos ∠ABC = Area of the shaded region
2( AB )( BC ) = area of sector OAB − area of △AOC
82 + 7 2 − 6 2 50° 1
=
2(8)(7) =  π × 252 × − × 25 × x × sin 50°  cm 2
 360° 2 
11 ∵ Area of the shaded region = 160 cm2
=
16 50° 1
∴ π × 25 × − × 25 × x × sin 50° = 160
2

360° 2
x = 11.8 (cor. to 3 sig. fig.)

54
Certificate Mathematics in Action Full Solutions 4B

∴ OC = 11.8 cm 28. (a)

27. (a) Consider △OAC.


By the cosine formula,
OA2 + OC 2 − AC 2
cos ∠AOC =
2(OA)(OC )
6 2 + 9 2 − 132
= Join AC.
2(6)( 9)
Consider △ABC.
13
= − AC 2 = AB 2 + BC 2 − 2( AB )( BC ) cos ∠ABC
27
∠AOC = 119° (cor. to 3 sig. fig.) AC = 9 2 + 8 2 − 2(9)(8) cos 60° cm

OA 2 + AC 2 − OC 2 = 73 cm
cos ∠OAC = = 8.54 cm (cor. to 3 sig. fig.)
2(OA)( AC )
6 2 + 132 − 9 2
= (b) Consider △ABC.
2(6)(13)
AB + BC + AC
31 Let s1 = .
= 2
39
9 + 8 + 8.544
∠OAC = 37.4° (cor. to 3 sig. fig.) ∴ s1 = cm
2
= 12.772 cm
(b) s1 − AB = (12.772 − 9) cm = 3.772 cm
s1 − BC = (12.772 − 8) cm = 4.772 cm
s1 − AC = (12.772 − 8.544) cm = 4.228 cm
Consider △ACD.
AC + CD + DA
Let s2 = .
2
8.544 + 7 + 6
∴ s2 = cm
2
Join OB. = 10.772 cm
∵ AC is the tangent to the circle at B. s2 − AC = (10.772 − 8.544) cm = 2.228 cm
∴ OB ⊥ AC s2 − CD = (10.772 − 7) cm = 3.772 cm
Consider △OAB. s2 − DA = (10.772 − 6) cm = 4.772 cm
OB Area of ABCD
sin ∠OAB =
OA = area of △ABC + area of △ACD
OB
sin 37.357° = = s1 ( s1 − AB )( s1 − BC )( s1 − AC ) +
6 cm
OB = 6 sin 37.357° cm s 2 ( s 2 − AC )( s 2 − CD )( s 2 − DA)
= 3.64 cm (cor. to 3 sig. fig.) = ( 12.772 × 3.772 × 4.772 × 4.228 +
10.772 × 2.228 × 3.772 × 4.772 ) cm 2
(c)
= 52.0 cm 2 (cor. to 3 sig. fig.)

29. (a) ∠ABC = 35° + 25° = 60°


∠ACB = 30° + 20° = 50°
∠BAC + ∠ABC + ∠ACB = 180° (∠ sum of △)
∠BAC + 60° + 50° = 180°
∠BAC = 70°
Consider △ABC.
With the notations in the figure, By the sine formula,
total area of the shaded regions AB BC
= area of △OAC − area of sector ODE =
sin ∠ACB sin ∠BAC
1 AB 12 cm
=  × 6 × 9 × sin 118.782 20° − =
2 sin 50° sin 70°
118.782 20°  2 12 sin 50°
π × 3.640 66 2 ×  cm AB = cm
360° sin 70°

= 9.78 cm (cor. to 3 sig. fig.)
= 9.93 cm 2 (cor. to 3 sig. fig.)

55
9 Basic Properties of CirclesMore about Trigonometry (II)

(b) Consider △BCD. ∵ Area of △ABC = area of △ADC


∠BDC + ∠DBC + ∠BCD = 180 (∠ sum of △) 1 1
× AB × BC × sin ∠ABC = × AD ×
∠BDC + 25° + 20° = 180° 2 2
∠BDC = 135° CD × sin ∠ADC
By the sine formula, 1 1
BD BC × 2 × 6 × sin θ = × x × 4 ×
= 2 2
sin ∠BCD sin ∠BDC sin(180° − θ )
BD 12 cm
= 6 sin θ = 2 x sin θ
sin 20° sin 135°
12 sin 20° x=3
BD = cm ∴
sin 135° AD = 3 cm
Consider △ABD.
AD 2 = AB 2 + BD 2 − 2( AB )( BD) cos ∠ABD (b) Consider △ABC.
2 2 AC 2 = AB 2 + BC 2 − 2( AB )( BC ) cos ∠ABC
 12 sin 50°   12 sin 20° 
  +  −
sin 70°   sin 135°  AC = 2 2 + 6 2 − 2( 2)(6) cos θ cm
AD =  cm
Consider △ACD.
 12 sin 50°  12 sin 20° 
2   cos 35 ° AC 2 = AD 2 + CD 2 − 2( AD)(CD ) cos ∠ADC
 sin 70°  sin 135° 
= 6.03 cm (cor. to 3 sig. fig.) AC = 32 + 4 2 − 2(3)(4) cos (180° − θ ) cm

∴ 2 2 + 6 2 − 2(2)(6) cos θ
30. Consider △ABD.
By the sine formula, = 32 + 4 2 − 2(3)(4) cos (180° − θ )
BD AB 40 − 24 cos θ = 25 + 24 cos θ
=
sin ∠BAD sin ∠ADB
48 cos θ = 15
BD sin ∠ADB
sin ∠BAD = 5
AB cos θ =
Consider △ADC. 16
By the sine formula,
DC AC
= sin θ = 1 − cos 2θ
sin ∠DAC sin ∠ADC
2
DC sin ∠ADC 5
sin ∠DAC = = 1 −  
AC (c)  16 
∠ADB + ∠ADC = 180° (adj. ∠s on st. line) 231
=
∠ADB = 180° − ∠ADC 256
sin ∠ADB = sin (180° − ∠ADC ) 231
=
= sin ∠ADC 16
∵ AD is the angle bisector of ∠BAC. Area of quadrilateral ABCD
= area of △ABC + area of △ADC
∠BAD = ∠DAC
= 2 × area of △ABC
sin ∠BAD = sin ∠DAC
1
BD sin ∠ADB DC sin ∠ADC = 2 × × AB × BC × sin ∠ABC
= 2
∴ AB AC  1 231  2
BD DC =  2× × 2× 6× cm
=  2 16 
AB AC  
BD AB = 11.4 cm 2 (cor. to 3 sig. fig.)
=
DC AC
32. Consider △PBA.
31. (a) Let AD = x cm. ∠BPA = 60° − 50°
∵ ABCD is a cyclic quadrilateral. = 10°
∴ ∠ABC + ∠ADC = 180° (opp. ∠s, cyclic quad.) By the sine formula,
∠ADC = 180° − θ AP BA
=
sin ∠PBA sin ∠BPA
AP 100 m
=
sin 50° sin 10°
100 sin 50°
AP = m
sin 10°

56
Certificate Mathematics in Action Full Solutions 4B

Consider △AQP. 35. (a)


∠PAQ = 60° − 40°
= 20°
∠APQ = 90° − 60°
= 30°
∠PAQ + ∠AQP + ∠APQ = 180° (∠ sum of △)
20° + ∠AQP + 30° = 180°
∠AQP = 130°
By the sine formula,
PQ AP
=
sin ∠PAQ sin ∠AQP
100 sin 50°
m With the notations in the figure, consider
PQ
= sin 10° △AFB.
sin 20° sin 130° AB
 100 sin 50°  tan ∠AFB =
PQ =  × sin 20°  m FB
 sin 10° sin 130°  4m
tan 60° =
= 197 m (cor. to 3 sig. fig.) FB
∴ The height of the church is 197 m. 4
FB = m
tan 60°
33. (a) Consider △DCE. = 2.31 m (cor. to 3 sig. fig.)
DE ∠GBF = 30°
sin ∠DCE =
CD
∠CBF = 90° − ∠GBF
DE
sin 20° = = 90° − 30°
CD
DE = 60°
CD =
sin 20° Consider △BFC.
By the cosine formula,
(b) FC 2 = BC 2 + FB 2 − 2( BC )( FB ) cos ∠CBF
2
4  4 
10 2 +   − 2(10) 
FC =  tan 60°   tan 60°  m
cos 60°
= 9.07 m (cor. to 3 sig. fig.)

(b) Consider △BFC.


Draw PQ ⊥ FE. Join QC. By the cosine formula,
The angle between CP and plane BCEF is ∠ FB 2 + FC 2 − BC 2
cos ∠BFC =
PCQ. 2( FB)( FC )
Consider △PCD. 2.30942 + 9.06862 − 10 2
CD =
cos ∠PCD = 2( 2.3094)(9.0686)
PC
DE ∠BFC = 107° (cor. to 3 sig. fig.)

cos 50° = sin 20°


PC 35. (a)
DE
PC =
sin 20° cos 50°
PQ = DE
Consider △PCQ.
PQ
sin ∠PCQ =
PC
DE
=
DE
sin 20° cos 50°
= sin 20° cos 50°
∠PCQ = 12.7° (cor. to 3 sig. fig.)
∴ The angle between CP and plane BCEF is Let E be the mid-point of DC. Join AE and BE.
12.7°. The angle between the planes ACD and BCD is
∠AEB.
34. Consider △ABD and △ABC.

57
9 Basic Properties of CirclesMore about Trigonometry (II)

AB = AB (common side) Consider △AFN and △AEC.


∠ABD = ∠ABC = 90° ( given) ∠FAN = ∠EAC (common angles)
BD = BC (given) ∠AFN = ∠AEC = 90°
∴ △ABD ≅ △ABC (SAS) ∠ANF = 180° − ∠FAN − ∠AFN
AD = AC = 180° − ∠EAC − ∠AEC
∵ AD = AC and DE = EC = 3 cm
= ∠ACE
∴ AE ⊥ DC
∴ △AFN ~ △AEC (AAA)
∵ BD = BC and DE = EC = 3 cm
AF AN
∴ BE ⊥ DC =
AE AC
Consider △ABD.
AF 1
AD 2 = AB 2 + BD 2 =
(Pyth. theorem) AE 2
AD = 10 2 + 8 2 cm 1
AF = AE
2
= 2 41 cm
155
Consider △ADE. = cm
2
AD 2 = AE 2 + DE 2
(Pyth. theorem) 155
FE = cm
AE = (2 41 ) 2 − 3 2 cm 2
Consider △BEF.
= 155 cm
BF 2 = BE 2 + FE 2 − 2( BE )( FE ) cos ∠FEB
Consider △BDE.
2
BD 2 = BE 2 + DE 2  155 
(Pyth. theorem) ( 55 ) 2 +   −
BE = 8 2 − 32 cm  2 
BF =   cm
= 55 cm  155  11 
2( 55 )  
Consider △ABE.  2  31 
  
BE
cos ∠AEB =
AE 155
= cm
55 2
=
155 Consider △ABF.
11 AF 2 + BF 2 − AB 2
= cos ∠AFB =
31 2( AF )( BF )
∠AEB = 53.4° (cor. to 3 sig. fig.)  155 
2
 155 
2

 +  − 10
2
∴ The angle between the planes ACD and 
 2   2 
BCD is 53.4°. =
 155  155 
2  
 2  2 
(b)
9
= −
31
∠AFB = 107° (cor. to 3 sig. fig.)
∴ The angle between the planes ANM and
BNM is 107°.

36. (a)

Let F be the mid-point of MN.


The angle between the planes ANM and BNM is
∠AFB.
∵ M and N are the mid-points of AD and AC Draw XG ⊥ AD.
respectively. GX = AB
∴ AM = MD and AN = NC = 15 m
∵ AD = AC YG = (30 − 10 − 10) m
∴ AM = AN = 10 m
∵ AM = AN and MF = FN Consider △YGX.
∴ AF ⊥ MN XY 2 = GX 2 + YG 2
(Pyth. theorem)
XY = 152 + 10 2 m
= 5 13 m

58
Certificate Mathematics in Action Full Solutions 4B

The total distance travelled by the man Consider △ABD.


= CX + XY + YA AB 2 = AD 2 + BD 2
= (10 + 5 13 + 10) m AD = 7 2 − 5 2 m
= 38.0 m (cor. to 3 sig. fig.)
= 24 m (Pyth. theorem)
(b)
AF = AP − FP
= AP − CR
= (5 − 4) m
=1m
Consider △ADF.
AF
sin ∠ADF =
AD
1m
=
With the notations in the figure, 24 m
JX = 10 m ∠ADF = 11.8° (cor. to 3 sig. fig.)
∠JXK = 25°
∴ The angle that the sheet makes with the
Consider △JXK.
horizontal ground is 11.8°.
JK
sin ∠JXK =
JX
(b)
JK
sin 25° =
10 m
JK = 10 sin 25° m
YL = JK
= 10 sin 25° m
The angle between the path XY and the
horizontal ground is ∠YXL.
Consider △YXL.
YL
sin ∠YXL =
XY
10 sin 25°
= With the notations in the figure, consider
5 13 △ABF.
∠YXL = 13.6° (cor. to 3 sig. fig.) AB 2 = AF 2 + BF 2
∴ The angle between the path XY and the (Pyth. theorem)
BF = 7 2 − 12 m
horizontal ground is 13.6°.
= 48 m
37. (a) Similarly, CF = BF = 48 m
Consider △BFC.
BF 2 + CF 2 − BC 2
cos ∠BFC =
2( BF )(CF )
( 48 ) 2 + ( 48 ) 2 − 10 2
=
2( 48 )( 48 )
1
= −
24
sin ∠BFC = 1 − cos2∠BFC
2
1
Let D be the mid-point of BC. With the = 1 −  − 
notations in the figure, the angle that the sheet  24 
makes with the horizontal ground is ∠ADF. 5 23
=
∵ AB = AC = 7 m and BD = DC = 5 m 24
∴ AD ⊥ BC
Area of the shadow of the sheet at noon
= area of △BFC
1
= × BF × CF × sin ∠BFC
2
1 5 23  2
=  × 48 × 48 × m
2 24 
= 24.0 m 2 (cor. to 3 sig. fig.)

(c) (i)

59
(Pyth. theorem)

9 Basic Properties of CirclesMore about Trigonometry (II)

HZ = YZ − YH
  1 
= 10 −  + 5  m
  tan θ 
 1 
= 5 − m
 tan θ 

With the notations in the figure, consider


△ADF.
AD 2 = AF 2 + DF 2

DF = ( 24 ) 2 − 12 m
= 23 m
GP = HX (Pyth. theorem)
= DF
= 23 m
GH = PX
RP = CF
= 48 m
Consider △GPR.
RP 2 = GR 2 + GP 2
(Pyth. theorem)
GR = ( 48 ) − ( 23 ) m
2 2

=5m
Consider △CRZ.
CR
tan θ =
RZ
4
RZ = m
tan θ
RY = RZ − YZ
 4 
= − 10  m
 tan θ 
Consider △APX.
AP
tan θ =
PX
5
PX = m
tan θ
YH = GH − GR − RY
 5  4 
= −5− − 10  m
 tan θ  tan θ 
 1 
= + 5 m
 tan θ 

60
Certificate Mathematics in Action Full Solutions 4B

Consider △XHY.
YX 2 = YH 2 + HX 2 (b) Consider △CAE.
CE
2 sin ∠CAE =
YX =  1 + 5  + ( 23 ) 2 m AC
 tan θ  6 cm
2 sin 65.485° =
1 AC
= 23 +  + 5  m
 tan θ  AC =
6
cm
Consider △XHZ. sin 65.485°
ZX 2 = HZ 2 + HX 2 (Pyth. theorem) Consider △ACB.
By the cosine formula,
2
 1  AB 2 + BC 2 − AC 2
ZX =  5 −  + ( 23 ) 2 m cos ∠CBA =
 tan θ 2( AB )( BC )
2
 1 
2
 6 
= 23 +  5 −  m 52 + 52 −  
 tan θ   sin 65.485° 
=
2(5)(5)
When YX = ZX, ∠CBA = 82.5° (cor. to 3 sig. fig.)
2 2
1 1 
23 +  + 5  23 +  5 −
=  39. (a)
 tan θ   tan θ 
1 1
+5 = 5−
tan θ tan θ
2
= 0
tan θ
θ = 90° or 270° (rejected)
∴ The shape of the shadow of the sheet
is not an isosceles triangle.
(ii) The area of the shadow is equal to the area
of △PQR. It is because the bases and the
heights of the two shadows are the same.
With the notations in the figure, let HQ = x m.
38. (a) The angle between AC and the horizontal Then QK = (600 − x) m.
ground is ∠CAE. Consider △HPQ.
∵ ∠ADB = ∠AEC = 90° HQ
tan ∠HPQ =
∴ BD // CE (corr. ∠s equal) PQ
∵ BD // CE and AB = BC = 5 cm xm
tan 65° =
∴ AD = DE (intercept theorem) PQ
Consider △AEC. x
PQ = m
AC 2 = AE 2 + CE 2 tan 65°
(Pyth. theorem)
AE = 10 2 − 6 2 cm Consider △KPQ.
= 8 cm QK
tan ∠KPQ =
AD = DE PQ
=
1 (600 − x ) m
AE tan 40° =
∴ 2 PQ
1
=  × 8  cm PQ =
600 − x
m
2  tan 40°
= 4 cm x 600 − x
Consider △DAE. =
tan 65° tan 40°
By the cosine formula, x tan 40° = 600 tan 65° − x tan 65°
EA2 = AD 2 + DE 2 − 2( AD )( DE ) cos ∠ADE 600 tan 65°
x =
EA = 4 2 + 4 2 − 2( 4)( 4) cos 40° cm tan 40° + tan 65°
= 2.7362 cm = 431.258
Consider △CAE. 600 − x = 168.742
CE
tan ∠CAE =
EA
6 cm
=
2.7362 cm
∠CAE = 65.5° (cor. to 3 sig. fig.)
∴ The angle between AC and the horizontal
ground is 65.5°.

61
9 Basic Properties of CirclesMore about Trigonometry (II)

Consider △HPQ.
HQ 40. (a) Consider △DOC.
sin ∠HPQ =
PH DC
tan ∠DOC =
431.258 m OC
sin 65° =
PH hm
tan 30° =
431.258 OC
PH = m
sin 65° h
= 476 m (cor. to 3 sig. fig.) OC = m
tan 30°
Consider △KPQ. = 3h m
QK Consider △DAC.
sin ∠KPQ =
PK DC
168.742 m tan ∠DAC =
sin 40° = AC
PK hm
168.742 tan 60° =
PK = m AC
sin 40° h
= 263 m (cor. to 3 sig. fig.) AC = m
tan 60°
h
= m
(b) Consider △APH. 3
AH Consider △COA.
tan ∠APH =
PH OC 2 + OA2 − AC 2
AH cos ∠COA =
tan 44° = 2(OC )(OA)
475.841 m
h 2
AH = 475.841 tan 44° m ( 3h ) 2 + 60 2 −  
 3
= 460 m (cor. to 3 sig. fig.) =
2( 3h )( 60)
Consider △BPK. h2
BK 3h 2 + 3600 −
tan ∠BPK = = 3
PK 120 3h
BK
tan 33° = h 2 + 1350
262.516 m =
45 3h
BK = 262.516 tan 33° m
= 170 m (cor. to 3 sig. fig.) (b) Consider △DBC.
DC
tan ∠DBC =
(c) BC
hm
tan 45° =
BC
h
BC = m
tan 45°
=hm
Consider △COB.
OC 2 + OB 2 − BC 2
cos ∠COB =
2(OC )(OB)
( 3h) 2 + 90 2 − h 2
=
With the notations in the figure, 2( 3h)(90)
RB = HK = 600 m
AR = AH − RH 2h + 8100
2
=
= AH − BK 180 3h
= (475.841 tan 44° − 262.516 tan 33°) m h 2 + 4050
=
= 289.034 m 90 3h
Consider △ABR.
AR (c) ∵ cos ∠COA = cos ∠COB
tan ∠ABR =
RB h 2 + 1350 h 2 + 4050
=
289.034 m 45 3h 90 3h
=
600 m ∴
2h 2 + 2700 = h 2 + 4050
∠ABR = 25.7° (cor. to 3 sig. fig.)
h 2 = 1350
∵ The angle of depression of B from A
= the angle of elevation of A from B h = 36.7 (cor. to 3 sig. fig.)
∴ The angle of depression of B and A is 25.7
°.

62
Certificate Mathematics in Action Full Solutions 4B

h 2 + 1350
cos ∠COA =
45 3h
(d)
1350 + 1350
=
45 3 ( 1350 )
∠COA = 19.5° (cor. to 3 sig. fig.)
∴ The compass bearing of B from O is N19.5
°E.

41. (a) The angle between DF and BF is ∠DFB.


Consider △BFG.
BF 2 = BG 2 + FG 2 (Pyth. theorem)
BF = 6 2 + 8 2 cm
= 10 cm
Consider △DFE.
DF 2 = DE 2 + EF 2 (Pyth. theorem)
DF = 6 2 + 8 2 cm
= 10 cm
Consider △DAB.
DB 2 = DA2 + AB 2
(Pyth. theorem)
DB = 8 2 + 8 2 cm
= 128 cm
Consider △DFB.
By the cosine formula,
BF 2 + DF 2 − DB 2
cos ∠DFB =
2( BF )( DF )
10 2 + 10 2 − ( 128 ) 2
=
2(10)(10)
9
=
25
∠DFB = 68.9° (cor. to 3 sig. fig.)
∴ The angle between DF and BF is 68.9°.

(b)

Let EG and FH intersects at N. Join FM and


MN.
The angle between FM and plane EFGH is ∠
MFN.
Consider △FGH.
FH 2 = FG 2 + GH 2
(Pyth. theorem)
FH = 8 2 + 8 2 cm
= 8 2 cm

63
9 Basic Properties of CirclesMore about Trigonometry (II)

∵ EFGH is a square.
FN = NH 2. Answer: C
1 ∠A + ∠B + ∠C = 180° (∠ sum of △)
= FH ∠A + 60° + 75° = 180°
∴ 2 (property of square)
 1  ∠A = 45°
=  × 8 2  cm
2  By the sine formula,
= 4 2 cm AC BC
=
MN = CH = 6 cm sin B sin A
Consider △MFN. AC BC
=
MN sin 60° sin 45°
tan ∠MFN =
FN AC sin 60°
=
6 cm BC sin 45°
=
4 2 cm 3
∠MFN = 46.7° (cor. to 3 sig. fig.) = 2
1
∴ The angle between FM and plane EFGH is
46.7°. 2
6
=
(c) 2

3. Answer : C
By the cosine formula,
AB 2 + BC 2 − AC 2
cos ∠ABC =
2( AB )( BC )
(3x ) 2 + x 2 − 7 2
cos 60° =
2(3x )( x )
1 10 x 2 − 49
Join CM. =
∵ DFB is an isosceles triangle. 2 6x 2
∴ FM ⊥ DB 3x = 10 x 2 − 49
2

∵ CDB is an isosceles triangle. x2 = 7


∴ CM ⊥ DB x = 7
The angle between the planes FDB and DBC is
∠FMC.
Consider △MFN. 4. Answer : D
∠FMN + ∠MFN + ∠FNM = 180° (∠ sum of △)
∠FMN + 46.686° + 90° = 180°
∠FMN = 43.314°
∠CMN = 90°
∠FMC = ∠FMN + ∠CMN
= 43.314° + 90°
= 133° (cor. to 3 sig. fig.)
∴ The angle between the planes FDB and
DBC is 133°. With the notations in the figure,
by the sine formula,
Multiple Choice Questions (p. 229) PR PQ
=
1. Answer: B sin ∠PQR sin ∠PRQ
DC = AB = 6 cm (opp. sides of // gram) 30 m 96 m
Consider △BCD. =
sin ∠PQR sin 100°
BC 2 + CD 2 − BD 2
cos ∠BCD = 30 sin 100°
2( BC )(CD ) sin ∠PQR =
96
8 2 + 6 2 − 12 2 ∠PQR = 17.924° or 162.076° (rejected)
=
2(8)(6)
∠RPQ + ∠PQR + ∠PRQ = 180° (∠ sum of △)
11
= − ∠RPQ + 17.924° + 100° = 180°
24
∠BCD = 117.280° ∠RPQ = 62.076°
∠ABC + ∠BCD = 180° ∠TPQ = ∠TPR + ∠RPQ
∠ABC + 117.280° = 180° = 55° + 62.076°
∠ABC = 63° (cor. to the nearest degree) = 117.1°
∴ The true bearing of Q from P is 117.1°.

64
Certificate Mathematics in Action Full Solutions 4B

7+4+5
5. Answer : B ∴ s = cm
2
= 8 cm
s − AB = (8 − 7) cm = 1 cm
s − AC = (8 − 4) cm = 4 cm
s − BC = (8 − 5) cm = 3 cm
Area of △ABC
= s ( s − AB )( s − AC )( s − BC )
= 8 × 1 × 4 × 3 cm 2
= 4 6 cm 2

With the notations in the figure, by the cosine 8. Answer: B


formula, Area of ABDE = area of △ACE − area of △BCD
AB 2 + AC 2 − BC 2 1
cos ∠BAC = = × AC × CE × sin ∠ACE −
2( AB )( AC ) 2
42 + 72 − 92 1
= × BC × CD × sin ∠BCD
2( 4)(7) 2
2 1
= − =  × ( 4 + 2) × (3 + 5) ×
7 2
∠BAC = 106.6° 1 
sin θ − × 4 × 3 × sin θ  m 2
∠FAC = 180° − ∠DAB − ∠BAC 2 
= 180° − 36° − 106.6° = 18 sin θ m 2
= 37° (cor. to the nearest degree) ∵ Area of ABDE = 14 m2
∴ The compass bearing of C from A is S37°W. 18 sin θ = 14
∴ 7
6. Answer : A sin θ =
9
∠ABC = 90° − 30° = 60° θ = 51° (cor. to the nearest degree)
∠BAC = 90° − 25° = 65°
Consider △ABC. 9. Answer : C
∠ACB + ∠BAC + ∠ABC = 180° (∠ sum of △) Consider △ACD.
∠ACB + 65° + 60° = 180° AD
cos ∠CAD =
∠ACB = 55° AC
By the sine formula, x
cos γ =
AC AB AC
=
sin ∠ABC sin ∠ACB AC =
x
AC 12 m cos γ
=
sin 60° sin 55° Consider △ABC.
12 sin 60° By the sine formula,
AC = m BC AC
sin 55° =
sin ∠BAC sin ∠ABC
3
12 × x
= 2 m BC cos γ
sin 55° =
sin α sin β
6 3 x sin α
= m BC =
sin 55° sin β cos γ

7. Answer : D
10. Answer : A
Consider △BDC.
By the cosine formula,
BC 2 = BD 2 + CD 2 − 2( BD )(CD ) cos ∠BDC
BC = 7 2 + (3 2 ) 2 − 2(7)(3 2 ) cos 45° cm
= 5 cm
Consider △ABC.
AB + AC + BC
Let s = .
2

Let N be the mid-point of DE.

65
9 Basic Properties of CirclesMore about Trigonometry (II)

Join MN and HN. Consider △HAK.


The angle between MH and plane CDEH is ∠MHN. HA2 = HK 2 + AK 2
MN = AD = 4 cm
HK = 32 − 2 2 cm (Pyth. theorem)
1
NE = DE = 5 cm
2
1 KJ = DC
=  × 10  cm
2  = 4 cm
= 5 cm The angle between the planes HBC and ABCD is ∠
Consider △NHE. HJK.
HN 2 = HE 2 + NE 2 Consider △HJK.
(Pyth. theorem)
HK
HN = 4 2 + 5 2 cm tan ∠HJK =
JK
= 41 cm 5
Consider △MHN. =
4
MN
tan ∠MHN = ∠HJK = 29° (cor. to the nearest degree)
HN
4 cm ∴ The angle between the planes HBC and ABCD
= is 29°.
41 cm
∠MHN = 32° (cor. to the nearest degree)
HKMO (p. 231)
∴ The angle between MH and plane CDEH is 32°. 1.
11. Answer : C
AD = BC = 4 cm
Consider △ACD.
AC 2 = AD 2 + CD 2
(Pyth. theorem)
CD = 6 2 − 4 2 cm
= 20 cm
The angle between CD and plane BCEF is ∠DCE.
Consider △DCE.
CE
cos ∠DCE = Join OA, OF, OB and OE.
CD
Let OA = y cm,
3 cm
= then OA = OB = OE = OF = y cm.
20 cm 360°
∠DCE = 48° (cor. to the nearest degree) ∠AOF =
6
∴ The angle between CD and plane BCEF is 48°. = 60°
Consider △AOF.
12. Answer : A 1
= × OA × OF × sin ∠AOF
2
1
Area of △AOF = y 2 sin 60° cm 2
2
3y2
= cm 2
4
Area of ABCDEF = 6 × area of △AOF
3y2
3 3 = 6×
4
Let J be the mid-point of BC.
y= 2
Joint HJ and JK.
AK = KD QP = BE
1 = 2 y cm
= AD
2 = 2 2 cm
1
=  × 4  cm Area of square PQRS = QP
2

2 
= ( 2 2 ) 2 cm 2
= 2 cm
∵ Area of square PQRS = X cm2
∴ X = (2 2 ) 2
=8

66
Certificate Mathematics in Action Full Solutions 4B

2. By the cosine formula, = 18


c 2 = a 2 + b 2 − 2ab cos ∠C s1 − AD = 18 − 9 = 9
c 2 = a 2 + b 2 − 2ab cos 60° s1 − AB = 18 − 10 = 8
c = a + b − ab
2 2 2 s1 − BD = 18 − 17 = 1
Consider △BCD.
a 2 + b 2 = c 2 + ab
BC + DC + BD
a b Let s2 = .
+ =P 2
b+c a+c
10 + 21 + 17
a ( a + c ) + b (b + c ) ∴ s2=
=P 2
( a + c)(b + c)
= 24
a 2 + ac + b 2 + bc s2 − BC = 24 − 10 = 14
=P
( a + c)(b + c) s2 − DC = 24 − 21 = 3
∵ c 2 + ab + ac + bc s2 − BD = 24 − 17 = 7
=P Area of the cyclic quadrilateral
( a + c)(b + c)
= area of △ABD + area of △BCD
b(a + c) + c(a + c )
=P = s1 ( s1 − AD)( s1 − AB)( s1 − BD) +
( a + c)(b + c)
(a + c)(b + c ) s 2 ( s 2 − BC )( s 2 − DC )( s 2 − BD)
=P
(a + c)(b + c ) = 18 × 9 × 8 ×1 + 24 ×14 × 3 × 7
P =1 = 36 + 84
= 120
3.

Consider △ABD.
By the cosine formula,
BD 2 = AD 2 + AB 2 − 2( AD )( AB ) cos ∠DAB
= 9 2 + 10 2 − 2(9)(10) cos θ
= 181 − 180 cos θ
∠BAD + ∠BCD = 180° (opp. ∠s, cyclic
∠BCD = 180° − θ
quad.)
Consider △BCD.
By the cosine formula,
BD 2 = BC 2 + DC 2 − 2( BC )( DC ) cos ∠BCD
= 10 2 + 212 − 2(10)( 21) cos (180° − θ )
= 541 + 420 cos θ
181 − 180 cos θ = 541 + 420 cos θ
∴ 600 cos θ = −360
3
cos θ = −
5
BD = 181 − 180 cos θ
∴ 3
= 181 − 180 − 
 5
= 17
Consider △ABD.
AD + AB + BD
Let s1 = .
2
9 + 10 + 17
∴ s1=
2

67

You might also like